USMLE Forum - Largest USMLE Community
good for those whos exam is near - rizowana - Printable Version

+- USMLE Forum - Largest USMLE Community (https://www.usmleforum.com)
+-- Forum: USMLE Forum (https://www.usmleforum.com/forumdisplay.php?fid=1)
+--- Forum: Step 1 (https://www.usmleforum.com/forumdisplay.php?fid=2)
+--- Thread: good for those whos exam is near - rizowana (/showthread.php?tid=340380)



0 - ArchivalUser - 09-22-2008

PHARM_CNS
Q: A 12yo patient was treated for a reaction to a bee sting, what drug provides the best coverage of sympathomimetic receptors?
A: Epinephirine(Alpha1,2 and Beta 1,2)

Q: A 57 yo heart failure pt develops cardiac decompensation, what drug will give you adequate perfusion of his kidneys as well as tx for his Hypotension
A: Dopamine

Q: A fellow passenger on a Carnival cruise ship looks pale and diaphoretic, what antimuscarinic agent would you give them?
A: scopolamine

Q: A group of pts are rushed into the ER complaining of excessive sweating, tearing, salivation, HA, N and V, muscle twitching, difficulty breathing and diarrhea. What drug would be the most effective immediate tx
A: Atropine pts are suffering from Cholinestrase inhibitor poisining(Nerve gas/Organophosphate poisining)

Q: As an Anes you want to use a depolarizing neuromuscular blocking drug on your pt, what do you use
A: Succinylcholine

Q: By what mechanism does this drug help
A: Prevents the release of Ca from SR of skeletal muscle

Q: Clonidine is the preferred sym pathomimetic tx of HTN in pts with renal disease, why??
A: Centrally acting alpha agonist, thus causing a decrease in central adrenergic outflow, spairing renal blood flow

Q: Cocaine casues vasoconstriction and local anesthesia by what mechanism
A: Indirect agonist, uptake inhibitor

Q: Cocaine shares is mechanism of action with what antidepressant
A: TCA

Q: Dobutamine used for the tx of shock acts on which receptors
A: Beta1 more than B2

Q: Guanethidine enhances the release of Norepi?
A: No, it inhibits the release of Nor Epi

Q: How does angiotensin II affect NE release?
A: It acts presynaptically to increase NE release.

Q: How does botulinum toxin result in respiratory arrest?
A: Prevents the release of ACh, which results in muscle paralysis.

Q: How does dantrolene work?
A: Prevents the release of calcium from the sarcoplasmic reticulum of skeletal muscle.

Q: How does NE modulate its own release? What other neurotransmitter has this same effect?
A: NE acts presynaptically on alpha-2 receptors to inhibit its own release.
A: ACh also acts presynaptically through M1 receptors to inhibit NE release.

Q: How would hemicholinium treatment affect cholinergic neurons?
A: Hemicholinium inhibits the transport of choline into the nerve, thus inhibiting formation of ACh.

Q: How would you reverse the effect of a neuromuscular blocking agent?
A: Give an antichloinesterase - neostigmine, edrophonium, etc

Q: If a patient is given hexamethonium, what would happen to his/her heart rate?
A: It would increase to ~ 100 beats/min. Both sympathetic and vagal stimulation would be knocked out, but the SA node has an intrinsic pace of 100 beats/min, which is normally checked by vagal stimulation.

Q: Isopoterenol was given to a patient with a developing AV block, why?
A: Stimulates beta adrenergic receptors

Q: Norepi feedbacks and inhibits the presynaptic receptor by what mechanism
A: Binding to the presynaptic alpha 2 release modulating receptors

Q: Reserpine will block the syntheis of this drug and but not its precursor.
A: Blocks Norepi, but not Dopamine

Q: These drugs acts indirectly by releasing strored catecholamines in the presynaptic terminal
A: Amphetamine and Ephedrine

Q: What anticholinesterase crosses the blood-brain-barrier?
A: physostigmine

Q: What antimuscarinic agent is used in asthma and COPD?
A: Ipratropium

Q: What antimuscarinic drug is useful for the tx of asthma
A: Ipratropium

Q: What are the classic symptoms of cholinesterase inhibitor poisoning (parathion or other organophosphates)?
A: Diarrhea, Urination, Miosis, Bronchospasm, Bradycardia, Excitation of skeletal muscle and CNS, Lacrimation, Sweating, and Salivation = DUMBBELS; also abdominal cramping

Q: What are the clinical indications for bethanechol?
A: Activates cholinergic receptors on bladder and bowel smooth muscle, alleviating post-op and neurogenic ileus and urinary retention.

Q: What are the clinical indications for neostigmine?
A: Post-op and neurogenic ileus and urinary retention, myasthenia gravis, and reversal of neuromuscular junction blockade (post-op) through anticholinesterase activity.

Q: What are the indications for using amphetamine?
A: narcolepsy, obesity, and attention deficit disorder (I wouldn't recommend this)

Q: What are the nondepolarizing neuromuscular blocking drugs?
A: Tubocurarine, atracurium, mivacurium, pancuronium, vecuronium, rapacuronium

Q: What are the phases of succinylcholine neuromuscular blockade?
A: Phase 1 = prolonged depolarization, no antidote, effect potentiated by anticholinesterase; Phase 2 = repolarized but blocked, an anticholinesterase is the antidote for this phase.

Q: What are two indirect acting adrenergic agonists?
A: amphetamine and ephedrine

Q: What beta 2 agonist will help your 21yo Astma pt?
A: Albuterol, tertbutaline

Q: What cholinergic inhibitor acts by directly inhibiting Ach release at the presynaptic terminal
A: Botulinum

Q: What cholinomimetic is useful in the diagnosis of Myasthenia Gravis
A: Edrophonium

Q: What cholinomimetics might your pt be taking for his glaucoma
A: Carbachol, pilocarpine, physostigmine, echothiophate

Q: What class of drug is echothiophate? What is its indication?
A: anticholinesterase
A: glaucoma

Q: What conditions would you use dantrolene?
A: In treatment of malignant hyperthermia, due to concomitant use of halothane and succinylcholine. Also in neuroleptic malignant syndrome, a toxicity of antipsychotic drugs.

Q: What drug is used to diagnose myasthenia gravis?
A: edrophonium (extremely short acting anticholinesterase)

Q: What drugs target this enzyme
A: Neostigmine, pyridostigmine edrophonium physostigmine echothiophate

Q: What effect would atropine have on a patient with peptic ulcer disease?
A: Theoretically it could be used to block the cephalic phase of acid secretion (vagal stimulation).

Q: What effect would atropine have on the preganglionic sympathetic activation of sweat glands? Would this person sweat?
A: None. No, because atropine would block the postganglionic muscarinic receptors involved in sweat gland stimulation.

Q: What enzyme is responsible for the breakdown of ACh in the synaptic cleft?
A: Acetylcholinesterase; ACh is broken down into choline and acetate.

Q: What enzyme is responsible for the degredation of Ach
A: Acetylcholine esterase

Q: What enzyme is responsible for the production of Ach from Acetyl CoA and Choline
A: Choline acetyltransferase

Q: What is the clinical utility of clonidine?
A: Treatment of hypertension, especially with renal disease (lowers bp centrally, so flow is maintained to kidney).

Q: What is the clinical utility of cocaine?
A: The only local anesthetic with vasoconstrictive properties.

Q: What is the difference between the affinity for beta receptors between albuterol/terbutaline and dantroline?
A: Dobutamine has more of an affintiy for beta-1 than beta-2, and is used for treating heart failure and shock. Albuterol and terbutaline is the reverse, and is used in treatment of acute asthma.

Q: What is the difference in receptor affinity of epinephrine at low doses? High doses?
A: Prefers beta's at low doses, but at higher doses alpha agonist effects are predominantly seen.

Q: What is the effect of epinephrine infusion on bp and pulse pressure?
A: Increased systolic and pulse pressure, decreased diastolic pressure, and little change in mean pressure.

Q: What is the effect of guanethidine on adrenergic NE release?
A: It inhibits release of NE.

Q: What is the effect of norepinephrine on bp and pulse pressure?
A: Increases mean, systolic, and diastolic bp, while there is little change in pulse pressure.

Q: What is the effect of TCA's on the adrenergic nerve?
A: They inhibit reuptake of NE at the nerve terminal (as does cocaine).

Q: What is the only depolarizing neuromuscular blocking agent?
A: Succinylcholine

Q: What is the receptor affinity and clinical use of isoproterenol?
A: It affects beta receptors equally and is used in AV heart block (rare).

Q: What makes this drug effective
A: It antagonizes Ach M receptors and decreases parasym (GI) rxn

Q: What nondepolorizing agents could you have used
A: Tubocurarine, atra-, miv-, pan-,ve-, rapacuronium

Q: What other substances regulate the Norepi nerve ending
A: Ach, AngiotensinII

Q: What other syndrome can this drug tx
A: Neuroleptic malignant syndrome

Q: What physiological effects was the Anes using Atropine to tx
A: SLUD (salivation, Lacrimation, urination, Defecation)as well as airway secretion, GI motility, acid secretions

Q: What reversal agent could a Anes give to reverse the effects of Atropine
A: Bethanechol, Neostigmine, physostigmine

Q: What side effect of using atropine to induce pupillary dilation would you expect?
A: Atropine would also block the receptors in the ciliary muscle, causing an impairment in accommodation (cycloplegia).

Q: What sympathomimetic would you not prescribe for hypotension in a pt with renal artery sclerosis.
A: Norepinephrine (Alpha1,2 and beta 1)

Q: What type of neurological blockade would hexamethonium create?
A: Hexamethonium is a nicotinic antagonist, and thus is a ganglionic blocker.

Q: What would be the effect on blood pressure with infusion of the alpha -2 agonist clonidine?
A: Initially vasoconstriction would increase bp, but then it acts on central alpha-2 receptors to decrease adrenergic outflow resulting in decreased bp.

Q: What would be the next drug that you would give and why
A: Pralidoxime, regenerates active cholinestrase

Q: Which antimuscarinic agents are used in producing mydriasis and cycloplegia?
A: atropine, homatropine, tropicamide

Q: Which drug increases Sys BP w/o affecting Pulse Pressure
A: Epinephrine

Q: Which of epi, norepi, or isoproterenol results in bradycardia?
A: Norepinephrine

Q: Which of the following would atropine administration cause? Hypothermia, bradycardia, excess salivation, dry flushed skin, or diarrhea
A: Dry flushed skin, due to inhibition of sympathetic post-ganglionic blockade on muscarinic receptors of sweat glands. All others are opposite of what would be expected.

Q: Which of these three drugs will cause a reflex bradycardia in your pt (Norepi, Epi, or Isoporterenol)
A: Norepinephrine

Q: Which receptors does phenylephrine act upon?
A: alpha-1 > alpha-2; used as a pupil dilator, vasoconstrictor, and for nasal decongestion

Q: While at a tail gait party, you bite into a sandwich that a yellow jacket is also enjoying. Knowing your allergy to this creature, what should you do?
A: Epinephrine to treat anaphylaxis. Also useful if you have open angle glaucoma, asthma, or hypotension.

Q: Why are albuterol and terbutaline effective in tx of acute asthmatic attacks?
A: These B-2 agonists cause respiratory smooth muscle to relax.

Q: Why does atropine dilate the pupil?
A: Blocking muscarinic receptors in the circular fibers of the eye, results in unopposed action of radial muscles to dilate.

Q: Why does NE result in bradycardia?
A: NE increases bp, which stimulates baroreceptors in the carotid sinus and the aorta. The CNS signals through vagal stimulation to decrease heart rate.

Q: Why is carbachol and pilocarpine useful in treatment of glaucoma?
A: They activate the ciliary muscle of the eye (open angle) and pupillary sphincter (narrow angle).

Q: Why is pyridostigmine effective in the treatment of myasthenia gravis?
A: As an anticholinesterase it increases endogenous ACh and thus increases strength.

Q: Why is reserpine effective in treating HTN?
A: Reserpine inhibits dopamine transport into vesicles, attenuating its conversion to NE by dopamine beta-hydroxylase.

Q: Why is there a drop in systolic, mean, and diastolic bp with infusion of isoproterenol?
A: Stimulating beta receptors stimulates heart rate, but beta receptor induced vasodilation reduces peripheral resistance.

Q: Why would a patient with cog-wheel rigidity and a shuffling gait be given benztropine?
A: Parkinson patients benefit from antimuscarinic agents through its inhibitory action within the indirect pathway.

Q: Why would dopamine be useful in treating shock?
A: Receptors = D1=D2>beta>alpha, thus increasing heart rate (beta) and blood pressure (alpha vasoconstriction) while maintaining kidney perfusion (dopamine receptors)

Q: Why would you give a drug like pancuronium or succinylcholine?
A: Useful in muscle paralysis during surgery or mechanical ventilation.

Q: Why would you use pralidoxime after exposure to an organophosphate?
A: Pralidoxime regenerates active cholinesterase.

Q: Will Hemicholinum affect the release of stored Ach during Cholinergic Stimulation
A: No, hemicholinum block the uptake of Choline and thus Ach synthesis

Q: Would blockade of muscarininc receptors in the bladder be useful in treating urinary retention?
A: No. Atropine is used to reduce urgency in mild cystitis. So it would aggravate the urinary retention.

Q: Would Hexamethonium be an effective substitute
A: No, hexamethonium targets Nicotinc receptors and will block Parasym, Sym, as well as Somatic systems

Q: You tx your pt with halothane as well and he has also developed malignant hypothermia, what drug can you give
A: Dantrolene

Q: Your patient develops a marked arrythmia due to a prolonged depolarization, can you tx this w/ Neostigmine
A: No cholinesterase inhibitors will potentiate the stimulating action of Succinlycholine

Q: Your patient has acute angle glaucoma, does this affect your tx
A: Yes, Scopolamine would antagonize his glaucoma

Q: Your patient wants an effective drug to treat his motion sickness, what would you prescribe
A: Scopolamine


SYNDROMES
SYNDROMES

ENVIRONMENTAL CAUSES
¢ ACUTE RADIATION SYNDROME: Radiation
exposure.
o 12 hours post-exposure: Vomiting
o 24 hours post-exposure: Prostration (extreme
exhaustion), fever, diarrhea
o Later: Petechial hemorrhage, hypotension,
tachycardia, profuse bloody diarrhea, maybe
death.
¢ CHINESE RESTAURANT SYNDROME: MSG
reaction ------> Chest Pain, burning sensation over
parts of body.
¢ BROWN-SEQUARD SYNDROME: Damage
(injury) to half of spinal cord ------> symptoms:
o Loss of pain and temperature sensation on
contralateral side of body.
o Loss of proprioception and discriminatory
touch on ipsilateral side of body.
CARDIOVASCULAR
¢ ADAMS-STOKES SYNDROME: Heart block, with
slow or absent pulse, often accompanied by
convulsions.
¢ BARLOW SYNDROME: Floppy Mitral Valve
Syndrome; Massive Mitral Valve Prolapse ------>
Late apical systolic murmur, systolic click, or both.
¢ EISENMENGER'S SYNDROME: Ventricular-
Septal Defect ------> Pulmonary hypertension and
cyanosis.
¢ FLOPPY-VALVE SYNDROME: Mitral
Incompetence due to myxomatous degeneration of the
leaflets.
¢ LERICHE'S SYNDROME: Occlusion of distal
aorta ------>
o Hip, thigh, and calf fatigue.
o Impotence
¢ BEHCET'S SYNDROME: Vasculitis ------>
secondary symptoms:
o Oral and genital ulcers
o Uveitis
o Optic atrophy
¢ SHOULDER-HAND SYNDROME: Pain in
shoulder and swelling in hand, sometimes occurring
after Myocardial Infarction.
¢ SICK SINUS SYNDROME: Chaotic atrial activity;
continual changes in P-Waves. Bradycardia,
alternating with recurrent ectopic beats and runs of
tachycardia.
¢ SUPERIOR VENA CAVA SYNDROME: Caused
by a tumor. Obstruction of SVC ------>
o Edema
o Engorgement of the vessels of face,
neck, and arms.
o Nonproductive cough
o Dyspnea
¢ TAKAYASU'S SYNDROME: Arteritis of the
Aortic Arch, resulting in no pulse. Seen in young
women.
¢ WOLF-PARKINSON WHITE SYNDROME:
ECG pattern of Paroxysmal Tachycardia.
o Short PR interval
o Delta wave = early QRS complex.
IATROGENIC (or Secondary to Medical Treatment)
¢ AFFERENT LOOP SYNDROME:
Gastrojejunal loop obstruction, proximal to a
gastrojejunostomy.
o Ingestion of food produces nausea, pain,
and duodenal distension.
¢ ASHERMAN'S SYNDROME: Adhesions
within the endometrial cavity, causing
amenorrhea and infertility.
o Adhesions probably were caused by
surgery.
¢ ULYSSES SYNDROME: Ill effects from
follow-up diagnostic tests following a falsepositive
screening test.
NEOPLASTIC (Malignant or Benign)
¢ CARCINOID SYNDROME: Carcinoid tumor
producing Bradykinin + Serotonin ------>
secondary symptoms:
o Cyanotic flushing
o Diarrhea
o Bronchial spasm
o Edema, ascites.
¢ CRONKHITE-CANADA SYNDROME: GIPolyps
with diffuse alopecia (hair-loss) and nail
dystrophy.
o May see protein-losing enteropathy and
malabsorption.
¢ GARDNER'S SYNDROME: Multiple
inherited tumors, hereditary dominant trait.
o Skull osteomas, Fibromas, Epidermoid
cysts
o Colonic polyposis (APC gene) ------>
predisposition to colonic
adenocarcinoma.
¢ LAMBERT-EATON SYNDROME:
Progressive proximal muscle weakness
secondary to a carcinoma.
¢ MEIGS' SYNDROME: Fibroma of ovary with
ascites and hydrothorax
¢ PANCOAST SYNDROME: Tumor near
pulmonary apex ------>
http://www.brain101.info/ 2
o Neuritic pain of chest and arm
o Muscle atrophy of the arm
o Horner's Syndrome (impaired cervical
sympathetics)
¢ PEUTZ-JEGHERS SYNDROME: Polyposis
(hamartomas) of small intestine
o Also see melanin pigmentation of buccal
mucosa and skin around mouth and lips
CONGENITAL
¢ CEREBELLAR SYNDROME: Congenital
Cerebellar Ataxia
¢ CERVICAL SYNDROME: Supernumerary C7 rib -
-----> Pressure on brachial plexus ------> pain
radiating over shoulder, arm, and forearm over C7
distribution.
¢ DANDY-WALKER SYNDROME: Obstruction of
Foramina of Magendie and Luschka in infants ------>
Hydrocephalus.
¢ DIGEORGE SYNDROME: Congenital absence of
3rd and 4th Branchial Arches (Thymus and Parathyroid
Glands) ------> secondary symptoms:
o No cell-mediated immunity ------> Frequent
viral and fungal infections
o Characteristic facial deformities
¢ DOWN SYNDROME: Trisomy 21. Mental
retardation, characteristic facial features, Simeon
crease in hand.
¢ FANCONI'S SYNDROME Type I: Bone-marrow
hypoplasia ------> refractory anemia, pancytopenia.
¢ EHLERS-DANLOS SYNDROME: Congenital
defect in collagen.
o Hyper-elasticity and friability of the skin.
o Hyperextensibility of the joints.
¢ FETAL ALCOHOL SYNDROME: Fetal
malformations, growth deficiencies, craniofacial
anomalies, limb defects.
¢ GOODPASTURE'S SYNDROME: Autoantibodies
against basement membranes ------>
Glomerulonephritis (kidney) and hemoptysis (lungs).
o Often, death by renal failure
¢ KLINEFELTER'S SYNDROME: Trisomy XXY ---
---> testicular atrophy, increase in gonadotropins in
urine.
¢ KLIPPEL-FEIL SYNDROME:
o Cervical vertebrate fused
o Congenital short neck, limited neck rotation
o Abnormalities of the brainstem and
cerebellum
o Low hairline.
¢ LESCH-NYHAN SYNDROME: Deficiency of
HGPRT (Hypoxanthine-Guanine Phosphoribosyltransferase
------>
o Hyperuricemia, uric acid kidney stones
o Choreoathetosis
o Mental retardation, autism, spastic
cerebral palsy
o X-Linked recessive
¢ MARFAN SYNDROME: Connective Tissue
disorder ------>
o Arachnodactyly: Abnormally long
digits and extremities
o Subluxation of lens
o Dissecting aortic aneurism
¢ POSTRUBELLA SYNDROME: Infantile
defects resulting from maternal Rubella infection
during first trimester.
o Microphthalmos, cataracts
o Deafness
o Mental retardation
o Patent ductus arteriosis, Pulmonary
arterial stenosis
¢ PRADER-WILLI SYNDROME: Short stature,
mental retardation, polyphagia with marked
obesity, sexual infantilism.
¢ RENDU-OSLER-WEBER SYNDROME:
Hereditary hemorrhagic telangiectasia.
¢ SUDDEN INFANT DEATH SYNDROME:
Unexplained death in sleeping infants.
¢ TURNER'S SYNDROME: XO monosomy.
o Dwarfism
o Webbed neck
o Valgus of elbow.
o Amenorrhea
¢ WILSON SYNDROME: Congenital defect in
Ceruloplasmin, leading to buildup of copper -----
-> mental retardation, cirrhosis, hepatolenticular
degeneration.
ENDOCRINE, REPRODUCTIVE
¢ AMENNORRHEA-GALACTORRHEA
SYNDROME: Non-physiologic lactation,
resulting from endocrinologic causes or from a
pituitary disorder.
¢ CONN'S SYNDROME: Primary
Hyperaldosteronism ------> muscular weakness,
hypertension, hypokalemia, alkalosis.
¢ CUSHING'S SYNDROME: Hypersecretion of
cortisol ------> secondary symptoms and
characteristics:
o Fatness of face and trunk with wasting
of extremities
o Buffalo hump
o Bone decalacification
o Corticoid diabetes
o Hypertension
¢ PREMENSTRUAL SYNDROME: Abnormal
sensation in breasts, abdominal pain, thirst,
headache, pelvic congestion, nervous irritability.
o Ocassionally nausea and vomiting.
http://www.brain101.info/ 3
¢ SHEEHAN'S SYNDROME: Post-partum pituitary
necrosis ------> hypopituitarism.
¢ STEIN-LEVENTHAL SYNDROME: Polycystic
ovary ------> infertility, amenorrhea, hirsutism. Seen
in obese women.
¢ TESTICULAR FEMINIZATION SYNDROME:
Insensitivity to Testosterone. Male
Psuedohermaphroditism
o Complete female external genatalia,
incompletely developed vagina, rudimentary
uterus.
PULMONARY
¢ KARTAGENER'S SYNDROME: Situs Inversus
(lateral transposition of lungs) resulting from chronic
sinusitis and bronchiectasis.
¢ HAMMAN-RICH SYNDROME: Interstitial fibrosis
of the lung.
¢ MIDDLE-LOBE SYNDROME: Chronic
pneumonitis and atalectasis of middle lobe of right
lung.
¢ CHURG-STRAUSS SYNDROME: Allergic
Granulomatous Angiitis: Asthma, fever, eosinophilia.
INFECTIOUS
¢ FITZ-HUGH-CURTIS SYNDROME: Gonococcal
Periphepatitis in woman, as a complication of
Gonorrhea.
¢ GUILLAN-BARRE SYNDROME: Infectious
Polyneuritis of unknown cause.
¢ HUNT'S SYNDROME: Herpe's Zoster infection of
Facial Nerve (CN VII) and Geniculate Ganglion ------
> facial palsy.
o Zoster of ear
¢ REYE'S SYNDROME: Loss of consciousness and
seizures in kids, after a viral infection treated by
aspirin.
¢ REITER'S SYNDROME: Symptom cluster.
Etiology is thought to be Chlamydial or postchlamydial.
o Urethritis
o Iridocyclitis (Conjunctivitis)
o Arthritis
o Skin lesions like karatoderma
blenorrhagicum
o Also can see fatty liver or liver necrosis.
¢ SCALDED SKIN SYNDROME: S. Aureus toxic
epidermal necrolysis.
¢ STEVENS-JOHNSON SYNDROME: Erythema
Multiforme complication.
o Large areas of skin slough, including mouth
and anogenital membranes.
o Mucous membranes: stomatitis, urethritis,
conjunctivitis.
o Headache, fever, malaise.
¢ TOXIC SHOCK SYNDROME: Caused by
superabsorbent tampons. Infection with Staph
Aureus and subsequent toxicity of exotoxin
TSST ------> systemic anaphylaxis.
o Fever, vomiting, diarrhea
o Red rash followed by desquamation
¢ WATERHOUSE-FRIEDRICHSON
SYNDROME: Meningeococcal Meningitis ------
> DIC, hemorrhagic infarct of adrenal glands ----
--> fulminant adrenal failure.
o Vomiting, diarrhea.
o Shock
o Extensive purpura, cyanosis, circulatory
collapse.
RENAL
¢ KEMMELSTIEL-WILSON SYNDROME:
Diabetic Glomerulosclerosis.
¢ BARTTER'S SYNDROME: Juxtaglomerular
Cell Hyperplasia ------> secondary symptoms:
o Hyperaldosteronism, Hypokalemic
Alkalosis, elevated renin and
angiotensin
o No hypertension.
o Compare to Conn's Syndrome
¢ FANCONI'S SYNDROME Type II: Renal
aminoaciduria, glycosuria, hypophosphaturia,
cysteine deposition, rickets.
¢ THORN'S SYNDROME: Salt-losing nephritis.
NEUROLOGICAL
¢ CARPAL-TUNNEL SYNDROME:
Compression of Median Nerve through the
Carpal Tunnel ------> pain and parasthesia over
distribution of Median N.
¢ FROIN'S SYNDROME: Block in CSF flow ----
--> xanthochromia (yellow discoloration) of
CSF.
¢ ACUTE-BRAIN SYNDROME: Delirium,
confusion, disorientation, developing suddenly in
a person that was previously psychologically
normal.
¢ GERSTMANN'S SYNDROME: Lesion
between occipital area and angular gyrus ------>
symptoms:
o Finger agnosia, Agraphia, acalculia
o Right-left disorientation
¢ HORNER'S SYNDROME: Loss or lesion of
cervical sympathetic ganglion ------>
o Ptosis, miosis, anhydrosis
o Enophthalmos (caved in eyes)
http://www.brain101.info/ 4
¢ KORSAKOFF SYNDROME: Loss of short-term
memory in chronic alcoholism, caused by
degeneration of mamillary bodies.
¢ RILEY-DAY SYNDROME: Familial dysautonomia.
GASTROINTESTINAL
¢ MALLORY-WEISS SYNDROME: Laceration of
lower end of esophagus from vomiting ------>
hematemesis. Often seen in alcoholics.
¢ MALABSORPTION SYNDROME: Impaired
absorption of dietary substance ------> diarrhea,
weakness, weight loss, or symptoms from specific
deficiencies.
¢ BARRET SYNDROME: Chronic peptic ulcer of the
lower esophagus, resulting in metaplasia of
esophageal columnar epithelium ------> squamous
epithelium.
¢ ZOLLINGER-ELLISOHN SYNDROME: Gastrinsecreting
tumor in pancreas ------> Severe peptic
ulcers, gastric hyperacidity.
¢ PLUMMER-VINSON SYNDROME: Esophageal
Webs, leading to dysphagis and atrophy of papillae of
tongue.
o Also see hypochromic anemia,
splenomegaly.
RETICULOENDOTHELIAL, HEMATOLOGIC
¢ BANTI'S SYNDROME: Chronic Congestive
Splenomegaly with anemia, caused by either Portal
Hypertension or Splenic Vein Thrombosis.
¢ BUD-CHIARI SYNDROME:
o ACUTE: Hepatic Vein Thrombosis ------>
Massive ascites and dramatic death.
o CHRONIC: Gradual hepatomegaly, portal
hypertension, nausea, vomiting, edema,
ulimately death.
¢ DUBIN-JOHNSON SYNDROME: Defect in
excretion of conjugated bilirubin ------> recurrent
mild jaundice. Buildup of direct builirubin in blood.
¢ CHIDIAK-HIGASHI SYNDROME: Abnormalities
in leukocytes with large inclusions.
¢ CRUVEILHIER-BAUMGARTEN SYNDROME:
Symptoms cluster:
o Liver cirrhosis
o Caput Medussae
o Venous hum and thrill
¢ FELTY'S SYNDROME: Rheumatoid Arthritis with
splenomegaly, leukopenia, anemia, and
thrombocytopenia.
¢ LOFFLER'S SYNDROME: Eosinophilia with
transient infiltrates in lungs.
UNCATEGORIZED
¢ YELLOW-NAIL SYNDROME: Stop growth
of nails ------> increased convexity, thickening,
and yellowing of nails.
o Found in Lymphedema, bronchitis,
chronic bronchiectasis.
¢ COSTOCHONDRAL SYNDROME: Pain in
chest with tenderness over one or more
costochondral junctions.
o Similar to Tietze's Syndrome but no
specific inflammation.
¢ TIETZE'S SYNDROME: Costochondritis.
Swelling and tenderness of the costal cartilege.
¢ MIKULICZ'S SYNDROME: Salivary and
lacrimal enlargement as seen in several diseases:
o Sarcoidosis
o Tuberculosis
o Leukemia
¢ MUNCHAUSEN SYNDROME: Malingering -
- fabrication of a clinically convincing disease by
an itinerant malingerer.
¢ PICKWICKIAN SYNDROME: Symptom
cluster
o Obesity
o Hypoventilation
o Somnolence
o Erythrocytosis
¢ RESTLESS LEGS SYNDROME: Need to
stretch legs at night before going to sleep; twitch
in legs causing insomnia.
¢ STRAIGHT BACK SYNDROME: Loss of
normal kyphosis of thoracic spine ------>
o Straight spine
o Ejection murmur
o Widened cardiac silouhette on x-ray
¢ SJÖGREN'S SYNDROME: Autoimmune
complex
o Keratoconjuctivitis Sicca (dry eyes and
mouth)
o Dryness of Mucous membranes
o Telangiectasias in face
o Parotid enlargement
PHARM_CVS
Q: ACE inhibitors- clinical use?
A: hypertension, CHF, diabetic renal disease

Q: ACE inhibitors- mechanism?
A: reduce levels of Angiotensin II, thereby preventing the inactivation of bradykinin (a potent vasodilator); renin level is increased

Q: ACE inhibitors- toxicity?
A: fetal renal damage, hyperkalemia, Cough, Angioedema, Proteinuria, Taste changes, hypOtension, Pregnancy problems, Rash, Increased renin, Lower Angiotensin II (CAPTOPRIL)

Q: Acetazolamide- clinical uses?
A: glaucoma, urinary alkalinization, metabolic alkalosis, altitude sickness

Q: Acetazolamide- mechanism?
A: acts at the proximal convoluted tubule to inhibit carbonic anhydrase. Causes self-limited sodium bicarb diuresis and reduction of total body bicarb stores.

Q: acetazolamide- site of action?
A: proximal convoluted tubule

Q: Acetazolamide- toxicity?
A: hyperchloremic metabolic acidosis, neuropathy, NH3 toxicity, sulfa allergy

Q: Acetazolamide causesÉ?
A: ACIDazolamide' causes acidosis

Q: Adenosine- clinical use?
A: DOC in diagnosing and abolishing AV nodal arrhythmias

Q: ADH antagonists- site of action?
A: collecting ducts

Q: adverse effect of Nitroprusside?
A: cyanide toxicity (releases CN)

Q: adverse effects of beta-blockers?
A: impotence, asthma, CV effects (bradycardia, CHF, AV block), CNS effects (sedation, sleep alterations)

Q: adverse effects of Captopril?
A: fetal renal toxicity, hyperkalemia, Cough, Angioedema, Proteinuria, Taste changes, hypOtension, Pregnancy problems, Rash, Increased renin, Lower Angiotensin II (CAPTOPRIL)

Q: adverse effects of Clonidine?
A: dry mouth, sedation, severe rebound hypertension

Q: adverse effects of ganglionic blockers?
A: severe orthostatic hypotension, blurred vision, constipation, sexual dysfunction

Q: adverse effects of Guanethidine?
A: orthostatic and exercise hypotension, sexual dysfunction, diarrhea

Q: adverse effects of Hydralazine?
A: nausea, headache, lupus-like syndrome, reflex tachycardia, angina, salt retention

Q: adverse effects of Hydrochlorothiazide?
A: hypokalemia, slight hyperlipidemia, hyperuricemia, lassitude, hypercalcemia, hyperglycemia

Q: adverse effects of Loop Diuretics?
A: K+ wasting, metabolic alkalosis, hypotension, ototoxicity

Q: adverse effects of Losartan?
A: fetal renal toxicity, hyperkalemia

Q: adverse effects of Methyldopa?
A: sedation, positive Coombs' test

Q: adverse effects of Minoxidil?
A: hypertrichosis, pericardial effusion, reflex tachycardia, angina, salt retention

Q: adverse effects of Nifedipine, verapamil?
A: dizziness, flushing, constipation (verapamil), nausea

Q: adverse effects of Prazosin?
A: first dose orthostatic hypotension, dizziness, headache

Q: adverse effects of Reserpine?
A: sedation, depression, nasal stuffiness, diarrhea

Q: Amiodarone- toxicity?
A: pulmonary fibrosis, corneal deposits, hepatotoxicity, skin deposits resulting in photodermatitis, neurologic effects, consitpation, CV (bradycardia, heart block, CHF), and hypo- or hyperthyroidism.

Q: antidote?
A: slowly normalize K+, lidocaine,
A: cardiac pacer, and anti-Dig Fab fragments

Q: Beta Blockers- CNS toxicity?
A: sedation, sleep alterations

Q: Beta Blockers- CV toxicity?
A: bradycardia, AV block, CHF

Q: Beta Blockers- site of action?
A: Beta adrenergic receptors and
A: Ca2+ channels (stimulatory)

Q: BP?
A: decrease

Q: BP?
A: decrease

Q: Bretyllium- toxicity?
A: new arrhythmias, hypotension

Q: Ca2+ channel blockers- clinical use?
A: hypertension, angina, arrhythmias

Q: Ca2+ channel blockers- mechanism?
A: block voltage dependent L-type Ca2+ channels of cardiac and smooth muscle- decreasing contractility

Q: Ca2+ channel blockers- site of action?
A: Cell membrane Ca2+ channels of cardiac sarcomere

Q: Ca2+ channel blockers- toxicity?
A: cardiac depression, peripheral edema, flushing, dizziness, constipation

Q: Ca2+ sensitizers'- site of action?
A: troponin-tropomyosin system

Q: Cautions when using Amiodarone?
A: check PFTs, LFTs, and TFTs

Q: class IA effects?
A: increased AP duration, increased ERP increased QT interval. Atrial and ventricular.

Q: class IB- clinical uses?
A: post MI and digitalis induced arrhythmias

Q: class IB- effects?
A: decrease AP duration, affects ischemic or depolarized Purkinje and ventricular system

Q: class IB- toxicity?
A: local anesthetic.
A: CNS stimulation or depression.
A: CV depression.

Q: class IC- effects?
A: NO AP duration effect.
A: useful in V-tach that progresses to V-fib
A: and in intractable SVT
A: LAST RESORT

Q: class IC- toxicity?
A: proarrhythmic

Q: class II- effects?
A: decrease the slope of phase 4, increase PR interval (the AV node is particularly sensitive)

Q: class II- mechanism?
A: blocking the beta adrenergic receptor leads to decreased cAMP, and decreased Ca2+ flux

Q: class II- toxicity?
A: impotence, exacerbation of asthma, CV effects, CNS effects, may mask hypoclycemia

Q: Class III- effects?
A: increase AP duration,
A: increase ERP,
A: increase QT interval,
A: for use when other arrhythmics fail

Q: class IV- clinical use?
A: prevention of nodal arrhythmias (SVT)

Q: class IV- effects?
A: decrease conduction velocity, increase ERP, increase PR interval

Q: class IV- primary site of action?
A: AV nodal cells

Q: class IV- toxicity?
A: constipation, flushing, edema, CV effects (CHF, AV block, sinus node depression), and torsade de pointes (Bepridil)

Q: classes of antihypertensive drugs?
A: diuretics, sympathoplegics, vasodilators, ACE inhibitors, Angiotensin II receptor inhibitors

Q: clinical use?
A: angina, pulmonary edema (also, erection enhancer)
A: (Nitroglycerine, Isosorbide Dinitrate)

Q: clinical use?
A: CHF, atrial fibrillation

Q: contractility?
A: increase (reflex response)

Q: contractility?
A: decrease

Q: contraindications?
A: renal failure, hypokalemia, pt on quinidine

Q: decrease Digitoxin dose in renal failure?
A: NO

Q: decrease Digoxin dose in renal failure?
A: YES

Q: Digitalis- site of action?
A: Na/K ATPase

Q: Digoxin v. Digitoxin: bioavailability?
A: Digitoxin>95%
A: Digoxin 75%

Q: Digoxin v. Digitoxin: excretion?
A: Digoxin=urinary
A: Digitoxin=biliary

Q: Digoxin v. Digitoxin: half life?
A: Digitoxin 168hrs
A: Digoxin 40 hrs

Q: Digoxin v. Digitoxin: protein binding?
A: Digitoxin 70%
A: Digoxin 20-40%

Q: ejection time?
A: decrease

Q: ejection time?
A: increase

Q: EKG results?
A: inc PR, dec QT, scooping of ST, and T wave inversion

Q: end diastolic volume?
A: decrease

Q: end diastolic volume?
A: increase

Q: Esmolol- short or long acting?
A: very short acting

Q: Ethacrynic Acid- clinical use?
A: Diuresis in pateints with sulfa allergy

Q: Ethacrynic Acid- mechanism?
A: not a sulfonamide, but action is the same as furosemide

Q: Ethacrynic Acid- toxicity?
A: NO HYPERURICEMIA, NO SULFA ALLERGY; same as furosemide otherwise

Q: Furosemide- class and mechanism?
A: Sulfonamide Loop Diuretic. Inhibits ion co-transport system of thick ascending loop. Abolishes hypertonicity of the medulla, thereby preventing concentration of the urine.

Q: Furosemide- clinical use?
A: edematous states (CHF, cirrhosis, nephrotic syndrome, pulm edema), HTN, hypercalcemia

Q: Furosemide- toxicity? (OH DANG)
A: Ototoxicity, Hypokalemia, Dehydration, Allergy (sulfa), Nephritis (interstitial), Gout

Q: Furosemide increases the excretion of what ion?
A: Ca2+ (Loops Lose calcium)

Q: HDL effect?
A: no effect

Q: HDL effect?
A: increase

Q: HDL effect?
A: moderate increase

Q: HDL effect?
A: increase

Q: HDL effect?
A: DECREASE

Q: how do we stop angina?
A: decrease myocardial O2 consumption by:
A: 1-decreasing end diastolic volume
A: 2- decreasing BP
A: 3- decreasing HR
A: 4-decreasing contractility
A: 5-decreasing ejection time

Q: HR?
A: increase (reflex response)

Q: HR?
A: decrease

Q: Hydralazine- class and mechanism?
A: vasodilator- increases cGMP to induce smooth muscle relaxation (arterioles>veins; afterload reduction)

Q: Hydralazine- clinical use?
A: severe hypertension, CHF

Q: Hydralazine- toxicity?
A: compensatory tachycardia, fluid retention, lupus-like syndrome

Q: Hydrochlorothiazide- clinical use?
A: HTN, CHF, calcium stone formation, nephrogenic DI.

Q: Hydrochlorothiazide- mechanism?
A: Inhibits NaCl reabsorption in the early distal tubule. Decreases Ca2+ excretion.

Q: Hydrochlorothiazide- toxicity? (hyperGLUC, plus others)
A: Hypokalemic metabolic alkalosis, hyponatremia, hyperGlycemia, hyperLipidemia, hyperUricemia, hyperCalcemia, sulfa allergy.

Q: Ibutilide- toxicity?
A: torsade de pointes

Q: K+- clinical use?
A: depresses ectopic pacemakers, especially in digoxin toxicity

Q: K+ sparing diuretics- clinical use?
A: hyperaldosteronism, K+ depletion, CHF

Q: K+ sparing diuretics- site of action?
A: cortical collecting tubule

Q: K+ sparing diuretics- toxicity?
A: hyperkalemia, endocrine effects (gynecomastia, anti-androgen)

Q: LDL effect?
A: moderate decrease

Q: LDL effect?
A: large decrease

Q: LDL effect?
A: moderate decrease

Q: LDL effect?
A: decrease

Q: LDL effect?
A: decrease

Q: loop diuretics (furosemide)- site of action?
A: thick ascending limb

Q: Mannitol- clinical use?
A: ARF, shock, drug overdose, decrease intracranial/intraocular pressure

Q: Mannitol- contraindications?
A: anuria, CHF

Q: Mannitol- mechanism?
A: osmotic diuretic- increase tubular fluid osmolarity, thereby increasing urine flow

Q: mannitol- site of action?
A: proximal convoluted tubule, thin descending limb, and collecting duct

Q: Mannitol- toxicity?
A: pulmonary edema, dehydration

Q: mechanism?
A: vasodilate by releasing NO in smooth muscle, causing and increase in cGMP and smooth muscle relaxation (veins>>arteries)
A: (Nitroglycerine, Isosorbide Dinitrate)

Q: mechanism?
A: inhibits the Na/K ATPase,
A: increasing intracellular Na+
A: decreasing the function of the Na/Ca antiport
A: causing an increase in intracellular Ca2+

Q: mechanism?
A: Na+ channel blockers. Slow or block conduction. Decreased slope in phase 4 and increased threshold for firing in abnormal pacemaker cells.

Q: Mg+- clinical use?
A: effective in torsade de pointes and digoxin toxicity

Q: MVO2?
A: decrease

Q: MVO2?
A: decrease

Q: name five in class II?
A: propanolol, esmolol, metoprolol, atenolol, timolol

Q: name four HMG-CoA reductase inhibitors.
A: Lovastatin, Pravastatin, Simvastatin, Atorvastatin

Q: name four in class IA.
A: Quinidine, Amiodarone, Procainamide, Disopyramide

Q: name four in class III.
A: Sotalol, Ibutilide, Bretylium, Amiodarone

Q: name three ACE inhibitors?
A: Captopril, Enalapril, Lisinopril

Q: name three calcium channel blockers?
A: Nifedipine, Verapamil, Diltiazem

Q: name three in class IB.
A: Lidocaine, Mexiletine, Tocainide

Q: name three in class IC.
A: Flecainide, Encainide, Propafenone

Q: name three in class IV.
A: Verapamil, Diltiazem, Bepridil

Q: name three K+ sparing diuretics?
A: Spironolactone, Triamterene, Amiloride (the K+ STAys)

Q: name two bile acid resins.
A: cholestyramine, colestipol

Q: name two LPL stimulators.
A: Gemfibrozil, Clofibrate

Q: Nifedipine has similar action to?
A: Nitrates

Q: preferential action of the Ca2+ channel blockers at cardiac muscle?
A: cardiac muscle: Verapamil>Diltiazem>Nifedipine

Q: preferential action of the Ca2+ channel blockers at vascular smooth muscle?
A: vascular sm. Mus.: Nifedipine>Diltiazem>Verapamil

Q: Procainamide- toxicity?
A: reversible SLE-like syndrome

Q: Quinidine- toxicity?
A: cinchonism: HA, tinnitus, thrombocytopenia, torsade de pointes due to increased QT interval

Q: Ryanodine- stie of action?
A: blocks SR Ca2+ channels

Q: selectivity?
A: slectively depress tissue that is frequently depolarized (fast tachycardia)

Q: side effects/problems?
A: tastes bad and causes GI discomfort

Q: side effects/problems?
A: expensive, reversible increase in LFTs, and myositis

Q: side effects/problems?
A: red, flushed face which is decreased by ASA or long term use

Q: side effects/problems?
A: myositis, increased LFTs

Q: side effects/problems?
A: DECREASED HDL

Q: Sotalol- toxicity?
A: torsade de pointes, excessive Beta block

Q: Spironolactone- mechanism?
A: competitive inhibirot of aldosterone in the cortical collecting tubule

Q: TG effect?
A: slight increase

Q: TG effect?
A: decrease

Q: TG effect?
A: decrease

Q: TG effect?
A: large decrease

Q: TG effect?
A: no effect

Q: thiazides- site of action?
A: distal convoluted tubule (early)

Q: toxicity?
A: tachycardia, hypotension, headache - 'Monday disease'
A: (Nitroglycerine, Isosorbide Dinitrate)

Q: toxicity?
A: nausea, vomiting, diarrhea, blurred vision, arrhythmia

Q: Triamterene and amiloride- mechanism?
A: block Na+ channels in the cortical collecting tubule

Q: Verapamil has similar action to?
A: Beta Blockers

Q: what two vasodilators require simultaneous treatment with beta blockers to prevent reflex tachycardia and diuretics to prevent salt retention?
A: Hydralazine and Minoxidil

Q: which diuretics cause acidosis?
A: carbonic anhydrase inhibitors, K+ sparing diuretics

Q: which diuretics cause alkalosis?
A: loop diuretics, thiazides

Q: which diuretics decrease urine Ca2+?
A: thiazides, amiloride

Q: which diuretics increase urine Ca2+?
A: loop diuretics, spironolactone

Q: which diuretics increase urine K+?
A: all except the K+ sparing diuretics Spironolactone, Triamterene, Amiloride

Q: which diuretics increase urine NaCl?
A: all of them

PHARMA_GEN.
Q: Acetaminophen has what two clinical uses and lacks what one clinical use of the NSAIDs?
A: Acetaminophen has antipyretic and analgesic properties, but lacks anti-inflammatory properties.

Q: Can Heparin be used during pregnancy?
A: Yes, it does not cross the placenta.

Q: Can Warfarin be used during pregnancy?
A: No, warfarin, unlike heparin, can cross the placenta.

Q: Does Heparin have a long, medium, or short half life?
A: Short.

Q: Does Warfarin have a long, medium, or short half life?
A: Long.

Q: For Heparin what is the 1. Structure 2. Route of administration 3. Onset of action 4. Mechanism of action
A: 1. Structure - Large anionic polymer, acidic
A: 2. Route of administration - Paranteral (IV, SC)
A: 3. Onset of action - Rapid (seconds)
A: 4. Mechanism of action - Activates antithrombin III

Q: Heparin continued: 5. Duration of action 6. Ability to inhibit coagulation in vitro 7. Treatment for overdose 8. Lab value to monitor 9. Site of action
A: 5. Duration of action - Acute (hours)
A: 6. Ability to inhibit coagulation in vitro - Yes
A: 7. Treatment for overdose - Protamine sulfate
A: 8. Lab value to monitor - aPTT (intrinsic pathway)
A: 9. Site of action - Blood

Q: For Warfarin what is the 1. Structure 2. Route of administration 3. Onset of action 4. Mechanism of action
A: 1. Structure - Small lipid-soluble molecule
A: 2. Route of administration -Oral
A: 3. Onset of action - Slow, limited by half lives of clotting factors
A: 4. Mechanism of action - Impairs the synthesis of vitamin K-dependent clotting factors

Q: Warfarin continued 5. Duration of action 6. Ability to inhibit coagulation in vitro 7. Treatment for overdose 8. Lab value to monitor 9. Site of action
A: 5. Duration of action - Chronic (weeks or months)
A: 6. Ability to inhibit coagulation in vitro - No
A: 7. Treatment for overdose - IV vitamin K and fresh frozen plasma
A: 8. Lab value to monitor - PT
A: 9. Site of action - Liver

Q: Is toxicity rare or common whith Cromolyn used in Asthma prevention?
A: Rare.

Q: List five common glucocorticoids.
A: 1. Hydrocortisone
A: 2. Predisone
A: 3. Triamcinolone
A: 4. Dexamethasone
A: 5. Beclomethasone

Q: Of the following: 1. Antigen recognition, 2. Proliferation, 3. Differentiation synthesis,
Q: 5. Tissue injury; Which sites do each of the following act upon?
Q: 1. Prednisone 2. Cyclosporine 3. Azathioprine 4. Methotrexate
A: 1. Prednisone - 2. Proliferation, 5. Tissue injury
A: 2. Cyclosporine - 2. Proliferation, 3. Differentiation synthesis
A: 3. Azathioprine - 2. Proliferation
A: 4. Methotrexate - 2. Proliferation

Q: Sites continued: 5. Dactinomycin 6. Cyclophosphamide
A: 5. Dactinomycin - 2. Proliferation, 3. Differentiation synthesis
A: 6. Cyclophosphamide - 2. Proliferation

Q: Sites continued: 7. Antilymphocytic globulin and monoclonal anti-T-cell antibodies 8. Rh3(D) Immune globulin 9. Tacrolimus
A: 7. Antilymphocytic globulin and monoclonal anti-T-cell antibodies - 1. Antigen recognition, 2. Proliferation, 3. Differentiation synthesis
A: 8. Rh3(D) Immune globulin - 1. Antigen recognition
A: 9. Tacrolimus - 4. Cytokine secretion.

Q: Secretion of what drug is inhibited by Probenacid used to treat chronic gout?
A: Penicillin.

Q: The COX-2 inhibitors (celecoxib, rofecoxib) have similar side effects to the NSAIDs with what one exception?
A: The COX-2 inhibitors should not have the corrosive effects of other NSAIDs on the gastrointestinal lining.

Q: What are are the Sulfonylureas (general description) and what is their use?
A: Sulfonylureas are oral hypoglycemic agents, they are used to stimulate release of endogenous insulin in NIDDM (type-2).

Q: What are five advantages of Oral Contraceptives (synthetic progestins, estrogen)?
A: 1. Reliable (Alkinalize urine(CO3) to remove more -Weak bases>acidify urine to remove more

Q: How do you treat coma in the ER (4)?
A: -Airway -Breathing -Circulation -Dextrose(thiamine &narcan) -ABCD

Q: In coma situations you rule out what (7)?
A: -Infections -Trauma -Seizures -CO -Overdose -Metabolic -Alcohol (IT'S COMA)

Q: List some specifics of lead poisoning(4)?
A: -A57Blue lines in gingiva& long bones -Encephalopathy & Foot drop -Abdominal colic / -Sideroblastic anemia

Q: List the specific antidote for this toxin: Acetaminophen
A: -N-acetylcystine

Q: List the specific antidote for this toxin: Amphetamine
A: -Ammonium Chloride

Q: List the specific antidote for this toxin: Anticholinesterases (organophosphate.)
A: -Atropine & pralidoxime

Q: List the specific antidote for this toxin: Antimuscarinic (anticholinergic)
A: -Physostigmine salicylate

Q: List the specific antidote for this toxin: Arsenic (all heavy metals)
A: -Dimercaprol, succimer

Q: List the specific antidote for this toxin: Benzodiazepines
A: -Flumazenil

Q: List the specific antidote for this toxin: Beta Blockers
A: -Glucagon

Q: List the specific antidote for this toxin: Carbon monoxide
A: -100% oxygen, hyperbaric

Q: List the specific antidote for this toxin: Copper
A: -Penicillamine

Q: List the specific antidote for this toxin: Cyanide
A: -Nitrate, hydroxocobalamin thiosulfate

Q: List the specific antidote for this toxin: Digitalis
A: -Normalize K+, Lidocaine, & Anti-dig Mab

Q: List the specific antidote for this toxin: Heparin
A: -Protamine

Q: List the specific antidote for this toxin: Iron
A: -Deferoxamine

Q: List the specific antidote for this toxin: Lead
A: -EDTA, dimercaprol, succimer, & penicillamine

Q: List the specific antidote for this toxin: Methanol & Ethylene glycol
A: -Ethanol, dialysis, & fomepizole

Q: List the specific antidote for this toxin: Methemoglobin
A: -Methylene blue

Q: List the specific antidote for this toxin: Opioids
A: -B51Naloxone / naltrexone (Narcan)

Q: List the specific antidote for this toxin: Salicylates
A: -Alkalinize urine & dialysis

Q: List the specific antidote for this toxin: TPA & Streptokinase
A: -Aminocaproic acid

Q: List the specific antidote for this toxin: Tricyclic antidepressants
A: -NaHCO3

Q: List the specific antidote for this toxin: Warfarin
A: -Vitamin K & fresh frozen plasma

Q: What are the products and their toxicities of the metabolism of ethanol by / alcohol dehydrogenase?
A: -Acetaldehyde -Nausea, vomiting, headache, & hypotension

Q: What are the products and their toxicities of the metabolism of Ethylene Glycol by / alcohol dehydrogenase?
A: -Oxalic acid -Acidosis & nephrotoxicity

Q: What are the products and their toxicities of the metabolism of Methanol by / alcohol dehydrogenase?
A: -Formaldehyde & formic acid -severe acidosis & retinal damage

Q: Which drug(s) cause this reaction: Adrenocortical Insufficiency
A: -Glucocorticoid withdrawal

Q: Which drug(s) cause this reaction: Agranulocytosis (3)?
A: -Cloazapine -carbamazapine -colchicine -PTU

Q: Which drug(s) cause this reaction: Anaphylaxis?
A: -Penicillin

Q: Which drug(s) cause this reaction: Aplastic anemia (5)?
A: -Chloramphenicol -benzene -NSAIDS -PTU -phenytoin

Q: Which drug(s) cause this reaction: Atropine-like side effects?
A: -Tricyclic antidepressants

Q: Which drug(s) cause this reaction: Cardiac toxicity?
A: -Daunorubicin & Doxorubicin

Q: Which drug(s) cause this reaction: Cinchonism (2)?
A: -Quinidine -quinine

Q: Which drug(s) cause this reaction: Cough?
A: -ACE inhibitors (Losartan>no cough)

Q: Which drug(s) cause this reaction: Cutaneous flushing (4)?
A: -Niacin -Ca++ channel blockers -adenosine -vancomycin

Q: Which drug(s) cause this reaction: Diabetes insipidus?
A: -Lithium

Q: Which drug(s) cause this reaction: Disulfram-like reaction (4) ?
A: -Metronidazole -certain cephalosporins -procarbazine -sulfonylureas

Q: Which drug(s) cause this reaction: Drug induced Parkinson's (4) ?
A: -Haloperidol -chlorpromazine -reserpine -MPTP

Q: Which drug(s) cause this reaction: Extrapyramidal side effects (3)?
A: -Chlorpromazine -thioridazine -haloperidol

Q: Which drug(s) cause this reaction: Fanconi's syndrome?
A: -Tetracycline

Q: Which drug(s) cause this reaction: Focal to massive hepatic necrosis (4)?
A: -Halothane -Valproic acid -acetaminophen -Amantia phalloides

Q: Which drug(s) cause this reaction: G6PD hemolysis(8)?
A: -Sulfonamides -INH -ASA -Ibuprofen -primaquine -nitrofurantoin /-pyrimethamine -chloramphenicol

Q: Which drug(s) cause this reaction: Gingival hyperplasia?
A: -Phenytoin

Q: Which drug(s) cause this reaction: Gray baby syndrome?
A: -Chloramphenicol

Q: Which drug(s) cause this reaction: Gynecomastia (6) ?
A: -Cimetidine -ketoconazole -spironolactone -digitalis -EtOH -estrogens

Q: Which drug(s) cause this reaction: Hepatitis?
A: -Isoniazid

Q: Which drug(s) cause this reaction: Hot flashes?
A: -Tamoxifen

Q: Which drug(s) cause this reaction: Neuro and Nephrotoxic?
A: -polymyxins

Q: Which drug(s) cause this reaction: Osteoporosis (2)?
A: -Corticosteroids -heparin

Q: Which drug(s) cause this reaction: Oto and Nephrotoxicity (3)?
A: -aminoglycosides -loop diuretics -cisplatin

Q: Which drug(s) cause this reaction: P450 induction(6)?
A: -Barbiturates -phenytoin -carbamazipine -rifampin -griseofulvin -quinidine

Q: Which drug(s) cause this reaction: P450 inhibition(6)?
A: -Cimetidine -ketoconazole -grapefruit juice -erythromycin -INH -sulfonamides

Q: Which drug(s) cause this reaction: Photosensitivity(3)?
A: -Tetracycline -amiodarone -sulfonamides

Q: Which drug(s) cause this reaction: Pseudomembranous colitis?
A: -Clindamycin

Q: Which drug(s) cause this reaction: Pulmonary fibrosis(3)?
A: -Bleomycin -amiodarone -busulfan

Q: Which drug(s) cause this reaction: SLE-like syndrome
A: -Hydralazine -Procainamide -INH -phenytoin

Q: Which drug(s) cause this reaction: Stevens-Johnson syn. (3) ?
A: -Ethosuxamide -sulfonamides -lamotrigine

Q: Which drug(s) cause this reaction: Tardive dyskinesia?
A: -Antipsychotics

Q: Which drug(s) cause this reaction: Tendonitis and rupture?
A: -Fluoroquinolones

Q: Which drug(s) cause this reaction: Thrombotic complications?
A: -Oral Contraceptives

Q: Which drug(s) cause this reaction: Torsade de pointes (2) ?
A: -Class III antiarrhythmics (sotalol) -class IA (quinidine)

Q: Which drug(s) cause this reaction: Tubulointerstitial Nephritis (5)?
A: -Sulfonamides -furosemide -methicillin -rifampin -NSAIDS (ex. ASA)
PHARM_ANTIMICROBIAL
Q: A common side effects of INF treatment is?
A: Neutropenia

Q: Antimicrobial prophylaxis for a history of recurrent UTIs
A: TMP-SMZ

Q: Antimicrobial prophylaxis for Gonorrhea
A: Ceftriaxone

Q: Antimicrobial prophylaxis for Meningococcal infection
A: Rifampin (DOC), minocycline

Q: Antimicrobial prophylaxis for PCP
A: TMP-SMZ (DOC), aerosolized pentamidine

Q: Antimicrobial prophylaxis for Syphilis
A: Benzathine penicillin G

Q: Are Aminoglycosides Teratogenic?
A: Yes

Q: Are Ampicillin and Amoxicillin penicillinase resistant?
A: No

Q: Are Carbenicillin, Piperacillin, and Ticarcillin penicillinase resistant?
A: No

Q: Are Cephalosporins resistant to penicillinase?
A: No, but they are less susceptible than the other Beta lactams

Q: Are Methicillin, Nafcillin, and Dicloxacillin penicillinase resistant?
A: Yes

Q: Clinical use of Isoniazid (INH)?
A: Mycobacterium tuberculosis, the only agent used as solo prophylaxis against TB

Q: Common side effects associated with Clindamycin include?
A: Pseudomembranous colitis (C. difficile), fever, diarrhea

Q: Common toxicities associated with Fluoroquinolones?
A: GI upset, Superinfections, Skin rashes, Headache, Dizziness

Q: Common toxicities associated with Griseofulvin are...?
A: Teratogenic, Carcinogenic, Confusion, Headaches

Q: Describe the MOA of Interferons (INF)
A: Glycoproteins from leukocytes that block various stages of viral RNA and DNA synthesis

Q: Do Tetracyclines penetrate the CNS?
A: Only in limited amounts

Q: Does Ampicillin or Amoxicillin have a greater oral bioavailability?
A: AmOxicillin has greater Oral bioavailability

Q: Does Amprotericin B cross the BBB?
A: No

Q: Does Foscarnet require activation by a viral kinase?
A: No

Q: Foscarnet toxicity?
A: Nephrotoxicity

Q: Ganciclovir associated toxicities?
A: Leukopenia, Neutropenia, Thrombocytopenia, Renal toxicity

Q: How are INFs used clinically?
A: Chronic Hepatitis A and B, Kaposi's Sarcoma

Q: How are Sulfonamides employed clinically?
A: Gram +, Gram -, Norcardia, Chlamydia

Q: How are the HIV drugs used clinically?
A: Triple Therapy' 2 Nucleoside RT Inhibitors with a Protease Inhibitor

Q: How are the Latent Hypnozoite (Liver) forms of Malaria (P. vivax, P.ovale) treated?
A: Primaquine

Q: How can Isoniazid (INH)-induced neurotoxicity be prevented?
A: Pyridoxine (B6) administration

Q: How can the t1/2 of INH be altered?
A: Fast vs. Slow Acetylators

Q: How can the toxic effects fo TMP be ameliorated?
A: With supplemental Folic Acid

Q: How can Vancomycin-induced 'Red Man Syndrome' be prevented?
A: Pretreat with antihistamines and a slow infusion rate

Q: How do Sulfonamides act on bacteria?
A: As PABA antimetabolites that inhibit Dihydropteroate Synthase, Bacteriostatic

Q: How do the Protease Inhibitors work?
A: Inhibt Assembly of new virus by Blocking Protease Enzyme

Q: How does Ganciclovir's toxicity relate to that of Acyclovir?
A: Ganciclovir is more toxic to host enzymes

Q: How does resistance to Vancomycin occur?
A: With an amino acid change of D-ala D-ala to D-ala D-lac

Q: How is Acyclovir used clinically?
A: HSV, VZV, EBV, Mucocutaneous and Genital Herpes Lesions, Prophylaxis in Immunocompromised pts

Q: How is Amantadine used clinically?
A: Prophylaxis for Influenza A, Rubella ; Parkinson's disease

Q: How is Amphotericin B administered for fungal meningitis?
A: Intrathecally

Q: How is Amphotericin B used clinically?
A: Wide spectrum of systemic mycoses: Cryptococcus, Blastomyces, Coccidioides, Aspergillus, Histoplasma, Candida, Mucor

Q: How is Chloramphenical used clinically?
A: Meningitis (H. influenza, N. meningitidis, S. pneumoniae), Conserative treatment due to toxicities

Q: How is Foscarnet used clinically?
A: CMV Retinitis in IC pts when Ganciclovir fails

Q: How is Ganciclovir activated?
A: Phosphorylation by a Viral Kinase

Q: How is Ganciclovir used clinically?
A: CMV, esp in Immunocompromised patients

Q: How is Griseofulvin used clinically?
A: Oral treatment of superficial infections

Q: How is Leishmaniasis treated?
A: Pentavalent Antimony

Q: How is Ribavirin used clinically?
A: for RSV

Q: How is Rifampin used clinically?
A: 1. Mycobacterium tuberculosis
A: 2. Delays resistance to Dapsone when used of Leprosy
A: 3. Used in combination with other drugs

Q: How is Trimethoprim used clinically?
A: Used in combination therapy with SMZ to sequentially block folate synthesis

Q: How is Vancomycin used clinically?
A: For serious, Gram + multidrug-resistant organisms

Q: How would you treat African Trypanosomiasis (sleeping sickness)?
A: Suramin

Q: In what population does Gray Baby Syndrome occur? Why?
A: Premature infants, because they lack UDP-glucuronyl transferase

Q: Is Aztreonam cross-allergenic with penicillins?
A: No

Q: Is Aztreonam resistant to penicillinase?
A: Yes

Q: Is Aztreonam usually toxic?
A: No

Q: Is Imipenem resistant to penicillinase?
A: Yes

Q: Is Penicillin penicillinase resistant?
A: No - duh

Q: IV Penicillin
A: G

Q: Mnemonic for Foscarnet?
A: Foscarnet = pyroFosphate analog

Q: MOA for Penicillin (3 answers)?
A: 1)Binds penicillin-binding proteins
A: 2) Blocks transpeptidase cross- linking of cell wall
A: 3) Activates autolytic enzymes

Q: MOA: Bactericidal antibiotics
A: Penicillin, Cephalosporins, Vancomycin, Aminoglycosides, Fluoroquinolones, Metronidazole

Q: MOA: Block cell wall synthesis by inhib. Peptidoglycan cross-linking (7)
A: Penicillin, Ampicillin, Ticarcillin, Pipercillin, Imipenem, Aztreonam, Cephalosporins

Q: MOA: Block DNA topoisomerases
A: Quinolones

Q: MOA: Block mRNA synthesis
A: Rifampin

Q: MOA: Block nucleotide synthesis
A: Sulfonamides, Trimethoprim

Q: MOA: Block peptidoglycan synthesis
A: Bacitracin, Vancomycin

Q: MOA: Block protein synthesis at 30s subunit
A: Aminoglycosides, Tetracyclines

Q: MOA: Block protein synthesis at 50s subunit
A: Chloramphenicol, Erythromycin/macrolides, Lincomycin, Clindamycin, Streptogramins (quinupristin, dalfopristin)

Q: MOA: Disrupt bacterial/fungal cell membranes
A: Polymyxins

Q: MOA: Unkown
A: Pentamidine

Q: MOABig Grinisrupt fungal cell membranes
A: Amphotericin B, Nystatin, Fluconazole/azoles

Q: Name common Polymyxins
A: Polymyxin B, Polymyxin E

Q: Name several common Macrolides (3)
A: Erythromycin, Azithromycin, Clarithromycin

Q: Name some common Sulfonamides (4)
A: Sulfamethoxazole (SMZ), Sulfisoxazole, Triple sulfas, Sulfadiazine

Q: Name some common Tetracyclines (4)
A: Tetracycline, Doxycycline, Demeclocycline, Minocycline

Q: Name the common Aminoglycosides (5)
A: Gentamicin, Neomycin, Amikacin, Tobramycin, Streptomycin

Q: Name the common Azoles
A: Fluconazole, Ketoconazole, Clotrimazole, Miconazole, Itraconazole

Q: Name the common Fluoroquinolones (6)
A: Ciprofloxacin, Norfloxacin, Ofloxacin, Grepafloxacin, Enoxacin, Nalidixic acid

Q: Name the common Non-Nucle

Report Abuse










0 - ArchivalUser - 09-22-2008

okt3 - 06/01/08 22:04

AST (SGOT) is increased in:

"A M-E-S-S-I-A-N-I-C L-A-M-B F-R-O-M God"

A-Alcoholic hepatitis: The AST / ALT ratio is
2:1 or greater.

M-Myocarditis
E-Eclampsia
S-Staphylococcal toxic shock syndrome
S-Seizures (muscle)
I-Isoniazid prophylaxis: monitor AST levels
A-Acetaminophen and Aspirin hepatotoxicity
N-certain Neoplasms
I-Intestinal injury (surgery, infarction)
C-Cirrhosis: AST exceeds ALT levels

L-Liver parenchyma: Acute viral hepatitis - The AST / ALT ratio is
usually less than 1.
L-Lyme disease, L-Lassa virus infection
A-Acute Pancreatitis Ransom criteria AST >250
M-acute Myocardial infarction
B-Burns

F-Fulminant liver failure as in Wilson's disease
R-Reye's syndrome
O-Occasionally: renal infarction
M-Muscle trauma

God-sGOT

___________________________________________________________

ALT (SGPT) is increased in:

"Nonalcoholic M-A-L-T"

N-Nonalcoholic steatohepatitis ALT > AST

M-Mononucleosis (infectious)
A-ALT: Alanine aminotransferase
L-Liver metastases
T~Viral hepatitis ALT > AST

___________________________________________________________

AST/ALT (SGOT/SGPT) ratio is increased in:

"Casting any Alternate CHAD Is In"

Casting - AST
Alternate - ALT

C-Cirrhosis
H-Hepatocellular carcinoma
A-Alcoholic hepatitis
D-Drug hepatotoxicity

Is-Intrahepatic cholestasis

In-Increased




0 - ArchivalUser - 09-22-2008

for learning ecg

http://student.bmj.com/issues/01/11/education/410.php




dreeman - 08/10/08 14:32

you sent sum ecg tracings bro....are they on boards now?
shud i give sum tim learning ecg?
Report Abuse





* Re:to irfan
#1416209
irfanmir - 08/10/08 14:51

my friend u should be able to know ECG, at least for WPW, MI atrial fib, ventricular fib, sinus tachy cardia, AV block , Digitalis toxicity, hyperkalemia. i hope fully post some good ECG Quiz next week. im at job right now its hard to answer in time sorry for answering late. GL
Report Abuse



* Re:to irfan
#1416216
analysis - 08/10/08 14:54

i had 3 ECG related q in my exam and ofcource the coomon one are those described above.



0 - ArchivalUser - 09-22-2008

ebnalfady - 09/05/08 21:13

if u want to add it is ok
Lewy bodies are round, eosinophilic inclusions that are x ic of Parkinson D.
Negri bodies are inclusions that are pathognomonic of rabies.
Hirano bodies are rodlike easinophilic incusions in Alzheimer D.
Neurofibrillary tangles are degnerated neurofilaments in Alzheimer D.
Cowdry Type A are intranuclear incusions that are found in herpes simples encephalitis
Intacytoplasmic inclusion bodies(NEGRI BODIES) - Rabies
Intranuclear inclusion bodies(COWDRY BODIES)- H.Simplex and V.Zoster Intracytoplasmic IB in variola - Guarnieri body
owel's eye body in cmv(cytomegalovirus)
Pick Disease
Some of the surviving neurons show a characteristic swelling (Pick cells) or contain Pick bodies, which are cytoplasmic, round to oval, filamentous inclusions that are only weakly basophilic but stain strongly with silver methods. Ultrastructurally, these are composed of straight filaments, vesiculated endoplasmic reticulum, and paired helical filaments that are immunocytochemically similar to those found in Alzheimer disease and contain 3 repeat tau. Unlike the neurofibrillary tangles of Alzheimer disease, Pick bodies do not survive the death of their host neuron and do not
Amyotrophic Lateral Sclerosis (Motor Neuron Disease)
neurons often contain Bunina bodies: PAS-positive cytoplasmic inclusions that appear to be remnant of autophagic vacuoles
G6PD deficiency
hienz bodyyyyyyyyyy

Mallory bodies


Acute alcoholic hepatitis is another potentially reversible form of liver injury Although fatty change is asymptomatic except for liver enlargement, alcoholic hepatitis can produce fever, liver tenderness, and jaundice. On histologic examination, there are focal areas of hepatocyte necrosis and cell injury manifest by fat accumulation and alcoholic hyalin, or Mallory bodies. Neutrophils accumulate around foci of necrosis


there are some bodies mark some diseaSES LIKE

ferruginous bodies. Sometimes asbestos


Sarcoidosis

Schaumann bodies and (2) stellate inclusions known as asteroid bodies enclosed within giant cells found in approximately 60% of the granulomas

psammoma bodies

Concentrically calcified structures termed psammoma bodies are often present within the lesion, usually within the cores of papillae. These structures are almost never found in follicular and medullary carcinomas, and so, when present, they are a strong indication that the lesion is a papillary carcinoma
Report Abuse



* Re:inclusion bodies
#1447845
ebnalfady - 09/06/08 11:09

Howell-Jolly body
Causes
Common causes of asplenia are splenectomy due to trauma, and autosplenectomy caused by sickle cell anemia. Other causes are radiation therapy involving the spleen, such as that used to treat Hodgkin lymphoma. Howell-Jolly bodies are also seen in: Severe hemolytic anemia, Megaloblastic anemia, Hereditary spherocytosis Myelodysplastic syndrome( MDS).







0 - ArchivalUser - 09-22-2008

Recent material from recent test-taker:

1. Drawing of urea cycle. Point to area where NH4 enters the cycle.
2. What causes RBCs to form aggregates in Sickle Cell Disease?
3. A gene product is thought to produce tRNA. What characteristic tells you that it is tRNA?
1. presence of many modified bases
2. 7 methyl guanine cap
3. polyA tail

4. Drawing of glycolytic, gluconeogenic, glycogen synthesis pathways. Point to the part of the drawing which is influenced by insulin.
5. G proteins and second messengers
6. Nitric oxide works thru which second messenger?
- cGMP
7. How would you distinguish the gene product translated from cDNA from that translated from ordinary DNA?

Some questions on glycogen storage diseases, lysosomal diseases. They were pretty straightforward. Study the pathways well.

Post-translational modifications, like capping and N-linked phosphorylation
Study also about lac operon, transcription, translation.

PHARMACOLOGY

This section is pretty ok except for those tracings. Antibiotics, the questions still tend to focus on the action of the drug at the molecular level. So For example, how does tetracycline work? Ans: by inhibiting attachment of tRNA to the ribosome. Something like that. Study also the AIDS drugs. As usual, what action of drug at the molecular level.
I didn't get a lot of anti-cancer drugs. Study adverse effects. Same with all drugs, study the common adverse effects. Drug for methotrexate
1. What drug is usually given as pre-treatment for leukemia chemotherapy?
- allopurinol
2. Drawing of nephron. Where does this diuretic act?
3. Which of the following anti-diuretics works by inhibiting the Na/K/2Cl transporter?
- furosemide

PATHOLOGY

1. Px with signs and symptoms of Goodpasture's Disease. What would LM show of a kidney biopsy?
-crescents
2. Photomicrograph of a heart muscle form a patient who died of MI. Date the infarct
3. Photomicrograph of plasma cells. Patient with lytic lesions in the vertebrae. What is the disease?
-multiple myeloma
4. Mechanism of I cell Disease
5. Px who is HIV positive with CD4 count less than 200. Px presents with bloody diarrhea. Colonoscopy showed reddish lesions with crypt abscesses. What is the associated pathology?
1. adenoCA
2. Kaposi's sarcoma
(I couldn't decide between these two )
6. 2 y.o. premature infarct dies after 2 days. Picture of blood clot in lateral ventricles. Cause?
1. SAH
2. Berry aneurysm
3. Intracranial bleed
7. 57 y.o. female with back pain. Xray showed lytic lesions. What is the underlying malignancy?
1. breast CA
2. thyroid CA
8. Px with Grave's disease. What is the visual field defect?
9. 16 y.o. girl with amenorrhea, shield-like chest. What is the karyotype?
-45 XO
10. Px with weakness, irritability, paresthesias. Photomicrograph showing basophilic stippling. To what was he exposed?
-lead
11. Photomicrograph of hypersegmented neutrophils, what is the deficiency?
- Vit B12 or folic acid

In general, pathology is OK. Classic presentations. It would be good to be an expert in patho to recover for hateful biochem. I got several photomicrographs of RBCs in Pxs with spherocytosis, sickle cell. Picture of WBCs, lymphocytes. Familiarize yourself with monocytes, basophils, you know

MICROBIOLOGY

1. 25 y.o male who went hiking in the woods 2 wks ago. He was treated with a penicillin drug for throat infection. PE shows linear erythematous rashes over arms, legs, hands. What is the cause?
- photosensitivity
- Varicella-Zoster
- Hypersensitivity
(I answered photosensitivity)
2. Px with inguinal lymphadenopathy, painless penile ulcer. What is the method of diagnosis?
-dark field microscopy
3. Px with annular erythematous rashes, migratory arthritis. Ab against which organism will be elevated in the Px's serum?
- Borrelia burgdorferi
4. Px who went on a trip to Brazil developed cardiomyopathy. What is the etiologic agent?
- Trypanosoma cruzi
- Leishmania braziliensis
5. ss (+) RNA, non-encapsulated virus that causes meningoencephalitis. To what group does it belong?
- Picornavirus
- Coronavirus
- Reovirus
- Etc
7. Px with greenish ear discharge, with a fruity smell. What is the antibiotic of choice?

As with the other sections, micro is fine except that you have to jog your memory fast coz you're under time pressure. viruses are tricky but luckily I just got one. Know the difference between gram (+) and G (-) bugs, virulence factors of bugs esp. cholera, pertussis bugs that act thru G proteins.

ANATOMY

1. Px with inattention, loss of drive. Point to the lesion. (Drawing of brain)
2. Px with injury to the knee. Point to the structure in the MRI that prevents excessive sliding of the tibia anteriorly.

3. Hypothenar atrophy and impaired adduction of fingers. What nerve is affected?
4. Px with difficulty doing push ups, doing wrist flexion, extension of fingers. What nerve is affected?

Study neuroanatomy well.

BEHAVIORAL SCIENCE

Lots of computations on odds ratio, relative risk, Hardy-Weinberg, values for sensitivity and specificity of two studies and you will be asked to compare. May questions also about biases. The study will be described and you have to determine what is the bias of study. Questions on alpha and beta errors. Same thing as before, the study will be described and you'll be asked what error is it. Questions on what is the best thing to say to a patient given a particular situation. For this you have to study ethics. Kaplan is OK

PHYSIOLOGY

Most of the questions on physiology involve charts, graphs then interpret it. Acid base balance in graph. You will also be given values of lung volumes and you'll be asked what disease the px has. Obstructive vs restrictive or both obstructive and restrictive? Values of pressures and oxygen content of each heart chamber, then you'll be asked what the pathology is - ASD, VSD, TOF etc.
Study also GI hormones, that's high yield. Graph on renal part of Kaplan that shows reabsorption curves in nephron. ( Na K Cl glucose PAH etc). They will point to a line and ask what substance that is. In my exam, high-yield are respiratory physio, renal, GI. Cardio: a lot of questions. Study the action potential of nerves and cardiac muscles well. Like ryanodine receptors

1.What structure removes Ca from cytoplasm after contraction?
- ryanodine receptors??
- microtubules
2.What is responsible for depolarization in SA node?
- influx of Ca









1 Pt alcholic with flapping tremors of hand Q) What part of brain is responsible for the tremors??
2 Pt with parkinsonism Q)Which gene is involved???
3 Pr with Nephorotic synd on dialysis develops peritonitis ? Q) Which bug is involved. S aures. Strep pneumo, e coli.
4 A graph of ventricular action potential Q) Quinidine act at which phase?
5 Pt taking drug for bipolar disorder develops hypothryoidisim.Q) which drug is responsible. Lithium, Carbazepine, gabapentin.
6 Pt on hiking trip develops diarrhea Q) they showed the pics of 5 bugs , which bug involved??
7 Which mechanisim involved in termination of action on NE and serotonin. A) Reuptake
8 Glucagon MOA Increase adenyl cylase or guanyl cyclase
9 Pt father died during anesthesia due to high fever.The child shd avoid which anesthesia? A Succinylcholine.
10 Pt with pseudomembranous colitis ....given metronidazole but the symptoms persisit. A Clostridium forms spores that perisist in Git
11 Child with hyaline membrane disease which enzyme deficient.?
12 Pt work in brake paddle industry develop restrictive lung disease.CXR shows B/L infiltratres. A Mesothelioma?
13 Dobutamine MOA? A increase ejection fraction.
14 Pt with blue sclera and multiple fractures which structure defective? A) Collagen
15 Stem cells present where in GIT??? krypts of liberkuhn, mucosa, lamina propria?
16 Pt on primaquine develops jaundice and dark urine.... A G6pd
17 Pt with microcytic hypochromic anemia.on eshophagoscopy a circumferential constriction observed ! A Plummervinson synd.
18 Pt with buccal pigmentation and multiple polyp.? A Peutz jegherz synd.
19 PT with acetaminophen toxicity ? Q) What is the treatment?
20 HBV virus Q) What is the MOA how does it cause Ca??
21 Aplastic anemia slide Q) what is the treatment A) Bonemarrow transplant.
22 Cystic fibrosis ??? Where is the prob. in golgi or ER while secreting CFTR?
23 Leuprolide MOA
24 Some crazy brand new drug for treatment of obesity???? i cant remember the name.
25 Diptheria toxin MOA?
26 Pt alcholic develps hypocalcemia??? how ??? A) pancreatic insufficiency, free fatty acid bind calcium?
27 oligohydramnios mother....baby has crushed face and limbs example of what????? syndrome , sequence.
28 CT scan of thorax. Q) mark the location of paravertebral sympathetic chain.
29 Pt with nisseria infection which complement deficient ??? c3, c4, c8
30 Pt with traumatic wound with crepitus. Q) Which bug?
31 Vincristine MOA
32 6 line scenario of scurvy.....which aminoacid is not hydroxylated A ) Proline.
33 Which MHC use TAP transport complex. I or 2
34 Pt with ataxia and telengictasia Q) what is the defect A) Dna repair enzymes.
35 Kaposi sarcoma causative bug?
36 Pt with rheumatoid arthritis develops renal failure..... slide show some thing green.... A) Renal amyloidosis
37 Women,obese and old develop left knee effusion and arthritis ? Dx??
38 Salicylate poisoning scenario what is the acid base disorder A) Resp Alkalosis and Metabolic acidosis
39 Mother and father suffering frm AR diseas. Both are affected and cousins. What is the chance that children will be affected? A) 100%
40 Female pt, sunbathing for 4 hrs.develps sunburn. Q)Which layer of skin affected? St corneum, epidermis,dermis.
41 Pt with homocysteinurea ? Q) Which aminoacid is increased in serum A) Methionine.
42 Pt with homocysteiurea? Q)which enzyme deficinet.
43 Mother 10wk pregnant order chorionic villus sampling baby increased risk of what??? NTD,Anencephaly?
44 Pt is french with prostrate ca cant speak english. Daugher offer to intrepret for her father. What is ur response. A)hire a professional french intrepreter.
45 Pt with h/o seizures and mental retardation.multiple lesions on the back. Q) pt increased risk of what in heart A)Cardiac rhabodymyoma
46 Pt in smoking cessation clinic .What shd doctor tell the pt regarding the advantage of quitting smoking. A)decrease risk of lung ca,copd, FEV becomes noramal after some months.
47 Pt with increased sr testosterone,testiculal atrophy,decreased sperm count,secondary sexual characteristics normal Q)dx A) Testerone intake.
48 Pt with insomnia,menstrual irregularities and headache-her mother has Alzehimers.she keeps waking her at night. Q) what is the treatment?
A)Tell her abt Alzheimer support grp
49 Pt, a child with ALL.Chemotherapy has 80% response.Pt parents deny treatment.What is ur response? A)Treat the child.
50 Old pt with COPD on respirator.Do not want to get hooked to respirator.Children of the pt insist that she use respirator.U decide to take off the respirator. Ur action is what legal and ethical,legal and unethical,illegal and unethical.
51 Which form of element is taken in for thyroid synthesis. Iodide or iodine.
52 Histology slide of small round blue cell, pain resistant to anelegesics. which genetic defect involved. A) Ewing sarcoma t(11:22)
53 Pt family has h/o colorectal ca deaths.Pt has multiple polyps. What is the treatment. Total colectomy,abdominal radiations.
54 pt with gastric ca has brown lesion in axilla Q) what is the lesion A)Seborrhea keratosis.
55 Pt with middiastolic rumbling murmur Q)mark the max intensity of murmur on the pic.
56 Slide of megaloblastic anemia Q)what is the treatment
57 Bordet gengau agar??? used for which bug.
58 Pt bitten by cat ?? which bug?? toxoplasma,bartonella and pasteurella were not in the options.....i was so pissed off.
59 Macrophages and APC first bring antigen and present at which location. A)draining lymphnodes.
60 Pt with increased PT, normal PTT and noramal BT which factor deficient.
61 Absiximab MOA
62 How ABO incompatiblity protect for Rh incomapatiblity..??
63 Pregnant mother taking tetracyline. Baby at risk of what??? Kernicterus,bone abnormalities.
64 Calcuation on alveolar ventilation.
65 ALS which gene defective.???
66 Pt with dilated airway till the edge of pleura. A)bronchiectasis
67 Pt with knife penetration above clavicle, lateral to sternoclavicular jt. It will penetrate what structure.??? A)Pleura, arch of aorta,brachiocephalic artery
68 PT with acromegaly, Drug of choice?? A) Octeriotide.
69 Pt with arthritis and b/l bells palsy A) Lyme disease.
70 Slide showing ectopic pregnancy!! the fetus was toooo small............
71 Pt diabetic and hypertensicve develop bloody diarrhea Q) Dx ischemic colitis.
72 If i yr child is never allowd to crawl and always carried by parents what developmental milestone will be underdeveloped. A) Gross motar.
73 Pt Hiv + develops sore throat,influenza like illness,generalized lymphadenopathy, Q)Dx A)latent period of hiv ,AIDS, pre AIDS syndrome.
74 Pt with osteoprosis what vitamin essential.?
75 Pt with senile osteoporosis Drug of choice. A)biphosphonates.
76 PT with h/o PID which structure like to contribute to risk of ectopic pregnancy. A)Fallopian tube.
77 Retinoic acid affect hooks to what gene A)HOX
78 PT with AIDS develop ring enhancing lesion on CT Dx ?? A)Toxoplasmosis
79 Female pt on straining discover a mass below the inguinal lig. Dx?? femoral hernia
80 Gross pic of APKD What is the inheritance??
81 Pt with +3 proeteinurea which cast cud be seen A)Fatty cast
82 Pt with b.p 200/150 Diabetic with eye problem. What is the cause of eye prob. A)Malignant htn or neovascularizatoin.
83 Age dependant changes CT will show what?? A) Increase size of ventricual cavity B/L
84 Women age 50+ ,hot flashes,vaginal dryness. labs will show what A)Increase FSH
85 Man work in airplane factory with loud noise present with b/l hearing loss? What is the pathology. A)loss of hair cells at the base,otosclerosis.
86 Pt on INH develops peripheral neuropathy Q)What vitamin deficient.
87 Pt with thyrotoxicosis? Q)which drug helpful to resolve acute symptoms.
88 Pt with lung lesion plus increase Calcium , h/o smoking which ca ? A) Sqamous cell ca
89 Familial hypercholestrolemia serum will show what??? A)increase ldl
90 MRA of circle of willis ....locate poster cerebral artery.
91 Pt with increase JVP,leg edema,dyspnea,orthopnea. Sr will show what ??? A) Increase sr BNP
92 Pt with holosystolic murmur at mitral area plus s3 and s4. develop atrial fibrillation. What impact will it have on murmur and heart sound.. A) murmur disappear,s4 disappear, murmur increase.
93 Child with urine dripping frm umbilicus what is the defect.?
94 Chest pain radiating to back in intersacpular region. A)Aortic dissection.
95 O2 saturation tracing of chambers of heart Q ) What is the defect A)VSD
96 Postpartum ischmia which organ most susceptible. A) ant pituitary,adrenal gland
97 Biconuate uterus ??? Which embryological structure is maldeveloped.
98 Pt with episodi HTN, catecholamine secreting tumor , most likely site.???? Adrenal medulla was not in the option... i pickd up sympathetic chain.
99 Pt come to u .and say that he had some kind of ezema for which he is taking meds from previous dr.. but the condition is worsening. u realize that it is not ezema but pityrisias rosea what is ur response???? A)previous dr made a mistake now i m changin meds.
100 Pt insist on taking chinese herbal med. what is ur response. A) no i will prescirbe only FDA approved drugs.




I must say that some of my answers might be wrong. if they are wrong just keep it to urself ...dont tell me.........
After i took the exam my dream of gettin 99 was shattered. I dont knw if i will be able to score good.I can only pray now.
I will rcv my result on 21st of may i think.Thnx to the forum!! it proved to a great source and i hope i m contributing something to this forum.
I will be gald to answer any questions..... if i dont respond on the forum then plz drop me an e mail... my email address is umair_y
I wish i cud meet everyone on the forum personally and one day perhaps our destiny will meet.I wish all the doctors best of luck!

1. SE of Cyclophosamide: h. cystitis
2. Tx of h. cystitis: MESNA
3. Tx of Pnumonia-
4. MOA of Gancyclovir-inhibibt thyimdate synthase?
5. MOA of Vincristine-inhibit microtubule formation at the M phase
6. MOA of Metotrexate-inhibit dihydrofolate reductase,used in Rheumatoid Arterheis
7. Prozosine...BPH, alpha1 antag, HTN
8. Action potential curve...Ca++ channels closed i dont know?
9. Cefataxime , indication-used in gonnorhea I think or is it chylamida
10. MOA of Glybride
11. Rifampin: tx for TB pt. along with 3 other drugs what are they?

12. Asked x drug (warferin) MOA: acts like anti vit-K drug(choice)

14. MOA of Colchicine-inhibit microtubule formation at the M phase
15. Tx for tonic clonic seizure?
16. Tx for a Kid with absent seizure beside ethosuximide?
17. Tx for a pt. with depression? Phenelzine
18. Furosemide : pick the correct area affected on the diagram of nephron.
19. Dopamin affects what NT? Ephineprine, NE, ...
20. What drug is given post MI? Aspirine
21. Tx for sleep apnea? methylamphedamine
22. MOA of Cocaine-inhibit reuptake of NE,EPI,DOPI
23. Ketoconazole?
24. MOA of AZT?
25. Tx for a pt with fever, productive cough, greenish sputum?
26. MOA of Rifampin? inhibits what assembly?
27. MOA and SE of Amphotrecine?
28. MOA of Gentamicine? Ristricts polysome formation

















1)) SQUAMOUS CELL CARCINOMA
- MALES
-CENTRAL LLOCATION
-KERATIN PEARLS
-INTRACELLULAR BRIDGES
-INVASIVE NESTS OF SQUAMOUS CELLS
-HISTOLOGICAL VARIETY WHCH CAVITATES
-MOST CMON LUNG CA WITH HYPERCALCEMIA
-OFCOURSE WHO CAN FORGET SMOKING

2))SMALL CELL CARCINOMA
-males
-smoking
-central locatio
-small round cells-em-cytoplasmic dense core neurosecretory granules
-clubbing least cmon in small cell
-paraneeoplastics r SIADH and lambert eaten and acth '
-worst prognosis
-most responsive to chemothereapy n radiotherapy
-mc histological variety to undergo mets
-SVS syndrome mmost cmonly associated
Report Abuse








LARGE CELL CARCINOMA
-gynecomastia most cmon

ADENOCARCINOMA
-mc lung ca worldwide
-peripheral
-scar carcinoma

BRONCHOALVEOLAR CARCINOMA
-subset of adeno
-terminal bronchioles or alveolar walls
-periphery
-columnar cells
-most cmon lung ca to mets to opp lung

yani - 02/15/07 13:37

pharmocology:
1. SE of Cyclophosamide: h. cystitis
2. Tx of h. cystitis: MESNA
3. Tx of Pnumonia
4. MOA of Gancyclovir
5. MOA of Vincristine
6. MOA of Metotrexate
7. Prozosine...BPH, alpha1, HTN
8. Action potential curve...Ca++ channels closed
9. Cefataxime , indication
10. MOA of Glybride
11. Rifampin: tx for TB pt.
12. Asked x drug (warferin) MOA: acts like anti vit-K drug(choice)
13. Identify Partial antagonist on the graph
14. MOA of Colchicine
15. Tx for tonic clonic seizure?
16. Tx for a Kid with absent seizure beside ethosuximide?
17. Tx for a pt. with depression? Phenelzine
18. Furosemide : pick the correct area affected on the diagram of nephron.
19. Dopamin affects what NT? Ephineprine, NE, ...
20. What drug is given post MI? I picked Aspirine
21. Tx for sleep apnea? methylamphedamine
22. MOA of Cocaine
23. Ketoconazole?
24. MOA of AZT?
25. Tx for a pt with fever, productive cough, greenish sputum?
26. MOA of Rifampin? inhibits what assembly?
27. MOA and SE of Amphotrecine?
28. MOA of Gentamicine? Ristricts polysome formation
Report Abuse

* Re:"2007 Crush Bank"
#660513
yani - 02/15/07 13:39

I agree, lets leave out all the thoughts and feelings about the exam and restrict this thread to SPECIFIC exam questions..........thought I'd add these in here - abrahem's translated exam questions (thanks to beauty and dreammind) -


1. A pt has orchitis and has decreased libido how do you treat him? (Testosterone, hCG, I cann™t see the other choices)

2- a man and a woman have autosomal doiminant disease got married, If you know that they have natural stature, which disease of the following, their children have a big chance to get it: cystic fibrosis b- achondroplasia c- MELAS.

3- what happened if the dashed part is affected (look at the diagram at Q3) many choices I choose loss of vibration an touch.

4- A girl got sore throat and was treated with penicillin without improvement, after sometime she died with heart problems, what is the cause? a. cocci b. CMV c. coxackie virus.

5- Viruses that carry virion associated polymerase what is the enzyme that is not found in the cytoplasm and this enzyme is used instead? a. DNA dependant DNA polymerase b. DNA dependant RNA polymerase c. RNA dependant RNA polymerase.

6- What do we need for PCR? a.deoxyribonuleotide b. DNA ligase.

7- How does mRNA exit the nuleouse? Transport dependant on ATP

8- What produce superoxide? a. superoxide dismutase b. NADPH oxidase c. catalase d. glutathione peroxidase

9- A boy has deficiency of skull mineralization what is affected? a. fibrillin b. chondroiin c. collagen d. heparin sulfate

10- A pt has kidney stones how do you treat him? a. mannitol b. furosemide c. spironolactone d. hydrochlorothiazide e. indapamide
11- Not clear
12- Not clear
13- MRI of brain with a mass in the ventricle this tumor most probably from? a. melanocyte b. epithelial cells c. meningioma
14- Q about apoptosis

15- A labeled draw of virus infecting a cell, asked about which level do the nucleoside RT inhibitors affect?
16- A pt had heart transplant and now is treated with cyclosporine, then he got respiratory problem with decreased lung functions what is the cause? a. CMV b. PCP
17- A pt has cardiac tamponade what do you see under microscope after two hours prove that it™s irreversible injury? A. acute swelling of mitochondria b. acute swelling of plasma membrane c. migration of nucleus
18- What is the most important factor for otitis externa? A. P.aeurginosa b. S. pneumoniae
19- A female with STD, she has microorganism covering skin cells and clue cells what is the diagnosis? A. candida b. trichomonas c. bacterial vaginosis
20- Two girls 16 and 14 years old came to you, the 14 years girl has STD what is your next step? A. treat her with permission of her older sister b. treat her without permission c. treat her with her parents permission
21- A pt with cancer in need to surgery, but his response is that œwhat is the benefit if I™m gonna die soon your response is? A. ask him to go and get help b. show him cases with the same disease so he can agree for surgery.



go here for suggested answers for this batch - http://usmleforum.com/showthread.php?tid=160377.php


I figure with even 5 questions a day, knowing the answers can make a significant difference in our scores Smile


Viruses that carry virion associated polymerase what is the enzyme that is not found in
the cytoplasm and this enzyme is used instead? a. DNA dependant DNA polymerase b. DNA
dependant RNA polymerase c. RNA dependant RNA polymerase

Re:"2007 Crush Bank"
#653475
ericvorheese - 02/09/07 19:35

great idea! here's my contribution...........an experience from a test that was taken earlier this week -


Biochem: some key enzyme deficiency ones (Lesch Nyhan, Maple Syrup Disease) but all of the questions were very obvious and did not require you to put much thought into it. Don't blow off porphyria and lead poisoning-- for some reason I got so many questions on that!

Molecular Bio: This was a big one! lots of questions about DNA regulation, transcription, translation, bacterial plasmids, etc. Sometimes these questions look very scary -- they are always so long and use long names for molecules or restriction enzymes that you have never heard of. You need to get used to the question style and realize that what they are asking is very simple. The NBME forms have questions very similar to the molecular bio ones I saw on the exam.

Pharm: Another one I thought would be difficult but not. Big drugs you should know a lot about (like antihypertensives, drugs for hyperlipidemias, cardic drugs, etc.) know side effects -- especially the chemo and immunosupressant drugs that kept on tripping me up so much.

Micro/Immuno: mostly bacterial processes and what you would use to treat them... or what was their mechanism of disease (i.e. endotoxn, exotoxin, etc.) Know immunology and cytokines well, as well as the functions of all the cells. Different immune deficiencies were all asked on my exam (there is one page in First Aid that sums them up very well).

Anatomy/Neuroanatomy: Always combined with a pathology question or an imagine. I had a few branchial plexus/lumbar plexus questions. Many questions would give you a clinical scenario, then ask you to identiy the artery/nerve/organ on a CT scan/MRI/angiogram/brain cross section.

Physio: This was almost always combined with Pathology--they would ask the physiology behind some path process. I had so many questions where the question asked "what would be the levels of x, y, and z enzymes/hormones?", answer choices being "increased, decreased, etc"

Pathology: lot of images -- MANY more than Qbank's representation.

images from NBME test were repeated on my actual exam

Goljan's book and lectures were great. He pointed out lots of things that wound up being on the exam and presents them in a way that really sticks. Also, his images are a great resource


GO 2007 CRUSH BANK!!!


















1... superior sagital sinus thrombosis
2....med. collatral injury
3....radial nerve injury
4....median nerve injury
5....pudendal nerve root value
6brain stem lesion
7...medulla lesion
8sub thal nuclei
9...type 2 collagene
PHYSIO-patho
1...hayline membrane dis
2....type II pneumocyte
3....pancr cancer
4....pheocro mo
5....choreo carcinoma
6 tuberous sclerosis
7...hyper trophic cardio myopathy
8 vsd
9AR
10...MI anterior descending artery block
11...hydronephrosis
12...H pylori
13...c peptide
14CAplan syndrome
15acromegaly
MICRO IMMUNO
1 herpis genit
2...HPV
3...dipth toxin
4....E coli PILI? fimbria
5 staph saprophy
6...strep pneumo
7 M TB
8....toxoplas
9.....microspo CANNi
10....H influenza
11...type 1 & 4 hyper sensitivity
12....wiskot aldrich
13....CGD
14....chemo taxis










USMLE Forum
Step 1
Step 2 CK
Step 2 CS
Matching & Residency
Step 3
Miscellaneous


Archives


Msgs Replies







>>

* pooled questions from previous ones
#247913
cancer123 - 11/30/07 20:20

pooled questions from previous ones



1)Pituitary tumor on visual fields. PITUTARY TUMOUR IS CRANIOPHRANGIOMA CAUSES BITEMPORAL HEMIANOPSIA.

2)Tumor anterior to temporal fossa affects wot?
3)Tends to fall to one side tongue deviates to other,
site of lesion? .THIS is problem in 12 cranial nerve . which is locted in lower medulla.
4)Pediatric infratentorial tumor. .most of the paediatric tumours are infra tentorial.they are celleblar, astrocytoma.2.medulloblastoma.3.gliomas

5)Midline neck mass at level of hyoid. .BRANCHIAL CYST

6)Esophagal hiatus of diaphragm. .IT OPENS AT THE LEVEL OF T10, IVC hiatus at T8
descending aorta opening at T12

7)Splenic venous drainage. .VENOUS SINUSOIDS IN CORDS OF BILROTH
8)Ambiguous genitalia. .AMBIGOUS GENITIAL is in female with excess of testosterone or in male with lack of testorone.

9)Melanosis coli. .MELONOSIS COLI I is due over use of laxatives.colon appers black in colour. Melanosis coli is due to use of laxatives not antibiotics. Usually anthraquinone ( senna and cascara)
10)Inhalational antigen stimulation leading to antibody
recovery from vaginal mucosa of experimental animal.
How? It is called "Dissemination of Immunologic Memory" , and it is the same mechanism used in Adenovirus Vaccine.
The adenovirus vaccine used by the military against adenovirus types 4 and 7 is an enteric-coated , live , non-attenuated virus preparation.
This vaccine produces an asymptomatic intestinal infection and thereby induces mucosal IgA memory cells.
These cells then populate the mucosal immune system throughout the body.
( Because Memory cells tend to home in a tissue-specific fashion , presumably returning to the type of tissue in which they first encountered antigen.)
Vaccine recipients are thus protected against adenovirus acquired by aerosol , which could otherwise produce pneumonia.

11)x-ray foot with calcification parallel to first
metatarsal with decreased dorsalis pedis. .myositis ossificens
12)Ligament that holds tibia from goin back: ant and posterior cruciate ligments---
that holds tibia from going back is Posterior crusiate ligament -----
ligament that prevents tibia from coming forward is Anterior cruciate ligament
13)Obturator nerve section: effects? obtutaror nerve injury leads to loss of adduction of thigh
14)Pelvic ascites, site of drainage on standing position. the site of drainage is vescicouteric pouch in females and rectovescical poch in males.
15)Aortic aneurysm rupture: effect on urinalysis in the
background of anuria. aortic aneurysm leads to renal infarction.so there will rbc in urine.
16)Hand decreased sensation lateral aspect. median nerve injury
17)Hand sensation ok, but extention of
metacarpophalangeal joints difficult. exention and flexion are done by lumbricals and interossis muscles. the extension at MP joint by--->extensors of the fingers
flexion at MP joint by--->lumbricals and introssei. so if extension is difficult means extensors are weak...which are supplied by radial nerve...

18)Clubbing description: choices bronchiectasis, asthma clubbing is seen in bronchiectasis

Q1) Cystic fibrosis heterozygous frequency?
Q2) Chemotherapy heart failure “ left heart/right
heart/myocardial fibrosis? Maybe Doxorubicin (Adriamycin) FA p.330 or Daunorubicin . But have no idea what is the mechanism
Q3) Stimulants and inhibitors of pancreatic secretions:
exocrine and endocrine.
Exocrine- Stimulants : CCK , Secretin , parasympathetics,
Inhibitors: None
Endocrine-
Insulin Stimulants : Glucose , Amino Acid , GIP , Glucagon,
Insulin Inhibitors : Somatostatin , Sympathetics (Alpha)
Glucagon Stimulants : Amino acid,
Glucagon Inhibitors : Insulin , Somatostatin

Q4) Cellular barrier between gall bladder cavity and
wall™s epithelial cells. is maybe asking for Tight junction. Tight junction must be there to protect the internal celluar structures from nasty bile
Q5) Embryology neural tube formation, cellular dna
fragmentation indicates? Apoptosis ?
Q6) War veteran describing killing business like “ defence mech?

Q7) Trigeminal nerve nucleus identification in medulla ct cross section.( you can just say verbally where its located)
Q8) Efferent renal constriction: effects on GFR &
Filtration fraction. GFR & FF both increases
Q9)Premature birth, surfactant and steroid therapy fails due to surface tension increase/decrease etc.
Q10) Barret esophageal cancer type? Adenocarcinoma , since barrett's is Metaplasia full of intesinal cells
Q11)GERD cuz of secretions from which type of gastric
cells? BRS Physiology(p.223) says that the relaxaton of lower esophageal sphincter is vagally mediated , and the neurotransmitter is VIP.
Or, maybe GERD could be due to gastric acid therefore , parietal cell could be an answer.
Q12) Cholecystectomy patients absorb fat from? Cholecystectomy pt. will still absorbing fat from terminal ileum I think, well ofcourse from small intestine cholecystectomy has nothing to do with the site of absorbtion .
the person still produces bile( in liver) just no storage or conecentration of bile.... thats why post op cholecystectomy patients are put on low fat diet
but site of absorbtion remains small intestine (ileum) as usual

Q13)Sputum silver staining for bugs. silver staining reminds me of two bugs,but I am not sure : Pneumocysyis carinii , Legionella pneumoniae.
But silver staining of PCP is done on pt.'s Bronchoalveolar lavage , I guess.
Q14) First thing to say to dad patient brought by 2
daughters who express wish to remain with patient.
Best statement to make to build rapport with patient
playing crossword on encounter. May be Doctor have to ask the pt. whether he wants their daughter in the room while interviewing
Q15) Post break-up with boyfriend, mother of patient finds synthroid tablets in nursing student daughter™s belongings. Doc™s response to patient? synthyroid is Thyroid hormone I guess, but not sure if it is T3 or T4. the pt. may be planning on weight loss because boyfriend hated her obesity .
q16)Fragile X family, parents concern regarding testing of ?normal 14-year daughter™s genome for similar problems.
Q 17) Girl child dumps classes since uncle™s death, and mother finds it helpful for daughter to be around home. Intervention required? Family therapy/psychotherapyetc.
Q18) Porphyrias: like the back of ur hands.
q19) Carbon tetrachloride: how it affects liver. CCl4 - The liver converts CCl4 to a CCl3 free radical , which damages the liver (fulminant hepatitis , fatty change) from Goljan's STARS pathology review p.93

Acyclovir mechanism of action. inhibhits DNA viral poymerase and phosphorylated by viral thymidine kinase
Q20) Hernia above n lateral to pubis. Hernia above the inguinal lig. is Inguinal hernia , and below the lig. is femoral hernia. So it could be both Indirect and direct inguinal hernia since it doesn't say anything about hernia's relation to inf. epigastric vessels.
Q21)Hydatidiform mole: learn how to interpret genotypes of moles.
Report Abuse

* Re:"2007 Crush Bank"
#681621
rumrum - 03/01/07 12:31

here are some more, which a few of us have complied, from various forms, and had to find the answers to some.....
you guys are awesome on this forum.. i have taken the exam once and didnt pass that was in 2004, now i am getting back into action to study for it once more... and no i am not indian...




Which 2 Joints are Never affected in RA ? Distal interphalangeal and the Back joints r never involved in RA
Q1) Best means of bringing down temperature of seizing
patient with meningitis: pharmacologic and otherwise. think non pharamacologic tt shd be hydrotherapy to bring down temp...n pharamacologic may be paracetamol by rectal route.
Q2) Acetaminophen overdose: how it damages. In overdose, the stores of GSH are depleted and and the metabolite N-acetylbenzoquinoneimine reacts with hepatocytes leading to liver necrosis: Acetaminophen forms free radicals in the liver and results in fulminant hepatitis/ renal papillary necrosis.
damages by free radicals which r removed by glutathione..N ACETYLCYSTEIN its antidote restores the glutathione levels
Q3) Antimicrosomal antibodies. involved in hashimoto's autoimmune thyroiditis and autoimmune hepatitis
Q4)Dialysis patient on transplant list gets angry often and misses dialysis appointments - intervention: need to spend more time with patient/discuss regarding
reconsideration of transplant listetc. Dialysis patient on transplant list gets angry often and misses dialysis appointments - intervention: need to spend more time with patient/discuss regarding reconsideration of transplant listetc. ??? need to spend more time with patient to know what exactly is his prob
Q5) Adenovirus “ structure? nonenveloped,ds linear DNA, icosahedral nucleocapsid, only virus with a fibre protruding from each of 12 vertices of teh capsid.
Q6) Brown fat of neonates. Function? site of heat production in a neonate---brown adipose tissue contain natural uncoupling protein, which allows energy loss in the form of heat to maintain basal temp
Q7) Longterm steroid use: effect on adrenals. adrenal suppression... therefore tapered slowly, neg feed back on ant pit so, decreased ACTH, so less stim of adrenalshypothalamo pituitary axis and eventually leads to atrophy of the adrenal gland.
Q8) Trypanosome cruzi: life cycle. try cruzi lifecycle involves reduvid bud as a vector and both humans and animals as reservoirs. life cycle. reservoirs are cats, dogs reduviid bug passes the trypomastigote as it bites and scratching implants in bite site.reservoir is rodents, armadillos. vector is reduviid bug. it transmits the trypo-mastigote form which turns into the amastigote form inside the body. At the bite site the lseion is called chagoma. Systemic symptoms are fever, lymphadenopathy . Tachycardia, meningoencephalitis. Then comes the intermediate phase where the patient is asymptomatic, with low levels of the parasite and antibodies against it. decades later-the chronic form emerges with dilated cardiomyopathy, megacolon, megaesophagus.Treatment is Nifurtimox.
Q9)Patient requiring dialysis says: don™t want machines to keep me alive. Doc™s next step?
Q10)Antifungals: big time. Drug interactions. Mechanism of actions.(lets write few antifungals with mech of action and main side effects...we will never forget it if we do so)
amphotericin B binds ergosterol creates pores.
flucytosine-inhibit thymidilate synthase.
azole-inhibit ergosterol synthesis
griesofulvin-interfere with microtubule function
terbinafine- inhibit squalene epoxide
DRUG INERACTIONS.
AZOLE GRP INCREASE TOXICITY OF FOLL BY INHIBITING ITS METABOLISM
bdz,cisapride,cyclosporine,fluoxetine,lovastatin,omeprazole
Q11) Alcoholic with nosebleed. Wot to expect in terms of AST, PT, and forgot( I dont know exactly whats being asked...lets see who figures it out) AST is increased, platelet count is decreased, prolonged PT
Q12)Picture of normal hand and a slender hand shown and asked for diagnosis.
Q13)Wernicke mammillary body identification on CT( here just name the structures affected in wernickes encephalopathy and ofcourse do know how to identify them...very very HY) mammillary bodies and dorsomedial nucleus of thalamus.Mamillary bodies on the ventral surface of the brain stem
Q14) How HBV causes liver cancer. HBV DNA integrates in host genomic DNA,MODIFICATIONOF GENE EXPRESSION. HBV INCORPORATES itself into the hepatocyte dna and triggers malignant growth. theres is a 200 X Risk of developing HCC in carriers versus noncarriers
Q15)Patient taking OCP. Smoker. Wot u wanna check first? smoking induces met enzymes increased metabolism of OCP'Sdecreased efficacy, Smoking itself is thrombogenic, and estrogen also is ask for history of thrombosis, may be check PT, PTT
Q16)Allopurinol “ effect on renal stones? It inhibits xanthine oxidase, decreasing uric acid
allopurinol used in pts of renal stones. as calcium oxalate stone formers are hyperuricemic
Q17)Radiation alopecia “ dermal histopathology? sparse mixed dermal inflammatory infiltrate consisting mainly of lymphocytes. . Mucinous deposits, confirmed by alcian blue/PAS stain, were observed within the epidermis of the hair follicle
Q18)Streptococcus pyogenes “ patient penicillin allergic.
Next step? Can give Clavulanic acid, Sulbactam, Tazobactam. Cephalosporins should be avoided in patients allergic to penicillin because of cross sensitivity.. Cephalosporins are susceptible to beta lactamase, but less than penicillin. ? penicillin allergic pts erythromycin or azithromycin are the alternatives, all GRP A Streptoccoci r susceptible to pencillin G, but neither Rhematic fever nor allergic pats benefit from pencillin treatment after onset. In pencillin allergies pats should get erythromycin or azithromycin.
.HOWEVER ENTEROCOCCAL ENDOCARDITIS CAN BE ERADICATED ONLY BY PENICILLIN OR VANCOMYCIN COMBINED WITH AN AMINOGLYCOSIDE"
Q19)t-Rna triplet codon function? The anti codon on the tRNA pairs with the codon for aa in mRNA
Q20) Nuclear membrane “ evolutionary importance between
eukaryotes and prokaryotes? all I know is nucleus membrane is absent in prokaryotes and present in euk may be evolutionary imp is to protect the genetic information...???

there are 4 trigeminal nuclei--->
1)primary sensory nuclei for sensations of face
2)spinal trigeminal nucleus for pain and tempreature sensation of face
3)Mesencepahalic nucleus for proprioception
4)primary motor nucleus for muscles of mastication



ZEBRA genes:Z EBV replication activator (ZEBRA) as lytic cycle markers.ZEBRA is one of the immediate early genes of EBV.it is considered to be expressed in the initial stage of the lytic cycle, before the viral particles have been dispersed.
Zebra sign:. Hemorrhage is often characterized by a typical, streaky bleeding pattern due to blood spreading in the cerebellar sulci. this zebra-pattern hemorrhage seems to be typical in a postoperative loss of CSF.
Zebra bodies - lipid inclusions with concentric lamellar structure with alternating light and dark-staining bands in neuronal cells in neimann picks
Zebra tumor = acoustic neuroma
Q1)Lead poisoning( symptoms) --Lead poisoning.. lead inhibits ferrochelatase and ALA dehydrase.., coproporhyrin and ALA accumulate in urine, symp”abdominal colic, sideroblastic anemia, wrist and foot drop, lead lines in gingivae and epiphysis of long bones, encephalopathy, basophilic stippling
Q2) Glutamine in urea cycle
Q3) Large intestine gross section: tumor shown. Type?
(felt like the napkin constriction)...I never heard of this kind of tumor...anyone knows?
Q4) Radiation therapy pelvis. Cause for urinary retention. ???cystits fibrosis
Q5) Osteogenesis imperfecta: defect? d? AD, mutation in collagen genes
Q6) Medial side of arm numbness after mastectomy: nerve involved? intercostobrachial n. also known as the lateral cutaneous br. of the ventral primary ramus of T2
Q7) Interleukins: big time. Learn function of each!!we can name some...and function.
IL1- stim produced mainly by macrophages
IL2 syn , pyrogenic, activates TH, IL2- downreg TH2, Tcell growth factor, stim Bcell
IL3- stim BM stem cells
IL4-class switch to IgE
IL5-Class swish to eosinophil, promote B cell prolif
IL6- produc of acute phase reactants
IL8-chemotactic and adhesion of neutrophils
IL10-Inhibit TH1, inhibit cytokine from macrophage
IL12-induce TH0 to TH1
Q8) Plummer Vinson: esophageal defect? Esophageal webs iron deficiency anemia,
o Stomatitis
o Glossitis
o Dysphagia
o Spoon-shaped nails
o Esophageal webs

Q9)Scurvy: enzyme/reaction involved? Prolyl and lysyl hydroxylase, the enzymes that catalyzes the hydroxylation, require vitamin C.
Q10)Reiter syndrome vs. Sjogren...
Reiter syndrome--- urethritis, conjunctivitis and ant uveitis, arthritis
Sjogren...Xerophthalmia, xerostomia, arthritis, parotid enlargement
Q11)Managing hypertensive crisis in MAO inhibitor patient on cheese: it™s pathogenesis? Treatment ot a Hypertensive Crisis from MAOI Reaction.
This is a medical emergency with cerebral vascular accidents being one of the greatest concerns. Medical consultation is warranted. Patients should report to an immediately to an emergency department. Initial treatment options include: Phentolamine 5 mg IV
or Thorazine 50 to 100 mg po
Care should be taken to ensure that the patient does not become dangerously hypotensive or that the hypertensive state returns once the intervention medication has worn off. Tyramine is actively transported into neurons and displaces NE, leading to intraneuronal release of NE. It can be degraded by MAO, but if you are on MAOI, its interaction will cause tyramine build up. Tons of NE will be replaced and hence the HTN crisis.
Q12) Neurotransmitter involved in Huntington. Decreased GABA and ACh
Q13)Aortic coarctation: site of constriction and symptoms. infantile type is preductal, Cardiomegaly, Pulmonary venous congestion , Right Ventricular Hypertrophy
Adult is postductal asso with notching of ribs, Displaced esophageal shadow rightward, Left Ventricular Hypertrophy, HT in UL, weak pulses in LL

Q14) Chronic antiplatelet drug in aspirin sensitive? Clopidogrel & Ticlopidine : They are back-up or alternative drugs to aspirin. They block ADP receptors on platelets , hence decrease activation
Q15)Stroke patient getting MI treated with streptokinase
dies due to cva. Cause? concluded that death of the patient occurred due to rebleeding on administration of streptokinase 1. Intracranial Hemorrhage :
As PRS has mentioned , the death could be due to intracranial hemorrhage. Since CVA refers to both ischemic (80%) and hemorrhagic(20%).
This is more likely than other explanation to me.

2. Therapeutic Failure :
It could be due to strptokinase's property itself.
Since most individuals have had a streptococcal infection sometime in their lives , circulating antibodies against streptokinase are likely to be present in most patients. These antibodies can combine with streptokinase and neutralize its fibrinolytic properties. (Therapeutic failure)
(lippincott Pharmacology 2nd edition p.203)
3. Property of all Thrombolytic agents
As the thrombolytic agents dissolute clot , Increased local thrombin may occur as the clot dissolves , leading to enhanced platelet aggregability and thrombosis. Strategies to prevent this include administration of antiplatelet drugs , such as aspirin , or antithrombotics , such as heparin.

Drugs that have been implicated in the development of serum sickness-like reactions include the following: allopurinol (Zyloprim), arsenicals and mercurial derivatives, barbiturates, captopril (Capoten), cephalosporins, furazolidone (Furoxone), gold salts, griseofulvin (Fulvicin, Grifulvin), halothane, hydralazine (Apresoline), iodides, methyldopa, para-aminosalicylic acid, penicillamine, penicillins, phenytoin (Dilantin), piperazine, procainamide (Procan SR, Procanbid, Pronestyl-SR), quinidine (Quinaglute, Quinalan, Quinidex, Quinora), streptokinase (Streptase, Kabikinase), sulfonamides, and thiouracils.

Q16)Leukotriene inhibitors? ZafirlukastLT receptor antagonist
Q17) Chromolyn sodium. Mechanism of action. prevent degranulation of mast cells
Q18)Grave™s pathophysiology. autoimmune TSH receptor antibodies, type II hypersensitivity
Q19) Couple in for infertility counseling. High BMI Wife
says, œit kills me not to be able to have a baby.
Doc™s best response? obese wife possibly has Polycystic Ovarian Syndrome (PCOS).
74% of PCOS pt. has infertility and PCOS accounts for 30% of overall infertility.
Therefore , the doctor may have to evaluate for PCOS by Lab. blood test and Ultrasonography. The lab blood test will show Increased androgen , LH , fasting insulin , prolactin and estrogens. The ultrasonography is the most sensitive diagnostic study.
If the Dx. of PCOS have been confirmed , Tx should be Weight reduction , quit smoking , and antiandrogen(because of hirsutism) and antiestrogen(clomiphen)

Q20)Baby awakens at night frightened, goes back to sleep.
Does not respond to questions b parents on that awakening. Best next step? The boy is possibly having Night Terrors. It occurs in Delta sleep , so you can't wake them and they can't remember what happened. And night terror is shown to be precursor to temporal lobe epilepsy.
The doctor's response should be either
examine for any epilepsy in the pt.(prevent or tx. the temporal lobe epilepsy) or ...
Identify & dealing with waking-time anxiety to relieve night terrors
celiac disease is the proximal bowel
and tropical sprue involves the entire bowel..

Q1) Prematue ejaculation. Best next step? Ans- selective seratonon reuptake inhibhitors like fluoxetine or even clorpramine (which is the most seratonin specific hetrocyclics)
Q2)Delirium: big time. ( just define)
Ans-Impaired conciousness(incontrast to dementia which is loss of memory and intellectual abilities)
causes-Huntingtons or parkinson's ds, CNS infection,trauma, systemic ds( like hepatic, cardiovascular), high fever substance abuse or withdrawal.
more common occurrence in children and elderly.
associated physical findings-autonomic dysfunction acute medical illness amd ABNORMAL EEG.
associated psycho findings-illusions hallucinations impaired conciousness sundowning(symptoms worse at night)
treatment- removal of underlying cause will allow symptoms to resolve
Q3)Malingering vs factitious. Ans-factitious.-when individual tries to simulate an illness for attention from medical personnel and can even undergo unnecessary medical and surgical procedures.
Malingering-when the same is done for financial or other obvious gain.
Q4)Leuprolide mechanism of action. ans- its a GnRH agonist and causes continued secretion of Gn from ant pituitary which causes the downregulation of Gn receptors--->inhibhition of FSH and LH--->supressed gonadal function.
used in treating precosious pubery, , prostatic Ca,endometriosis, premenopausal breast Ca,utrine leomyoma, PCOD,....can also be used as contraceptive in both males and females
Q5) Thromboangitis obliterans.( discuss in afew lines) Ans-also called buerger"s ds.affects most commonly smokers..it affects the whole neurovascular bundle of tibial poplteal or radial arteries.
the thrombus is composed of neutrophil containing micro abscesses.Pts frequently have raynaud's phenomenon
Q6)Lack of T tubules in muscles lead to? I think lack of T tubules in muscle will affect the membrane depolarization and hence contraction.
Q7)Tryptophan/tyrosine metabolism both of them are both ketogenic and glucogenic amino acids
Q8) phosphofructokinase “ inhibitors? ATP and citrate
Q9)Lysosomal storage disorders...just name and imp charecteristic
ans-fabry's and hunters are XR rest all are AR.
Tay sachs-def of hexosaminidase and GM2 gangliosides accumulate.
fabry's - def of alfa galactosidase and ceramide trihexoside accumulates.
Mertachromatic leuko dystrophy- def of arylsulfatase and sulfatides accumumalate
krabbes- beta galactosidase deficient and galactocerebroside accumulates
Nieman pick-sphingomylinase def and sphingomylin accumulates.
gaucher-def of beta glucosidase and glucocerebroside accumulates
Q10).S-100 positive slide, prognostic factor? S100 is a tumor marker for malig. melanoma and imp factor which determines prognosis is depth of the lesion
Q11)Gaucher bone marrow. Ans-gauchers cells present which are macrophages with wrinkeled paper cytoplasm
Q12)Young boy with breast biopsy for post-traumatic swelling showing multilayered cells lining ducts: next step?
maybe this is case of gynecomastia...its mostly noticed by medical personnel after trauma...In most cases, no treatment is needed,gynecomastia goes away in less than 3 years.Occasionally, medicines may be used to treat gynecomastia, especially if tenderness is a problem. treatment includes testosterone gel or surgery
Q13)S shaped bug, curved bug. curved is vibrio...does S shape also include in this genus Im not sure
Q14) Boy and pet dog both having diarrhea. No
ova/parasites. ?
q15) Scleroderma: cause for pulmonary hypertension? Ans-PH is high blood pressure in the arteries which take blood between the heart and lungs. When PH occurs along with other lung, heart, or systemic connective tissue disease (such as scleroderma), it is called Secondary Pulmonary Hypertension.when its due to scleroderma the cause maybe fibrosis
Q16) Pulmonary hypertension patient listed for lung transplantation. Pharmacologic management during waiting period?
maybe diuretics and oxygen therapy.
looked up a website which stated epoprostenol(FLOLAN) for those listed for lung transplant
Q17) Vascular bed, blood flow increased, howz capillary
flow controlled? this occurs by arteriolar constriction to maintain a constant flow in capillaries.
Q18) Carcinoid lung....(some imp features) Bronchial carcinoid tumors arise from Kulchitsky cells (argentaffin cells) within the bronchial mucosa.These cells are neurosecretory cells,which belong to APUD sysyem.They have the capacity to synthesize serotonin,ACTH,norepinephrine, bombesin, calcitonin, antidiuretic hormone (ADH), and bradykinin.
The clinical manifestations of bronchial carcinoids may arise because of their endobronchial or central location, their potential for metastatic spread, or their ability to produce vasoactive amines.
Hemoptysis is common, occurring in at least 50% of patients. This finding reflects the vascular nature of these lesions.
patients may present with complications due to the neurosecretory activity of the tumors. Bronchial carcinoids may secrete ACTH in quantities sufficient to cause Cushing syndrome in 2% of patients
Q19)Common cause for cataract and aged skin wrinkles?
Q20) Retinoblastoma, risk for another cancer? ans-Some, especially those in which the tumor involves both the eyes, are at a risk for developing other tumors like osteogenic sarcoma....maybe due to the radiation therapy..
but I also had read some where that retinoblastoma patients are at increased risk of developing some brain tumors( cant reme which specific brain tumor)...plz correct me if Im wron
1.Testicular CA drains to ? para aortic LN
2.Meckel™s is a remnant of? vitellointestinal duct
3.A child with tumor near 3rd vent will present with what symp? obstructive hydrocephalus
4.A pic of carotid arteriogram with arrow pointing to one branch, asked for symp in pt due to its block MI/angina symptoms
5.MRI of abd given, asked to identify structure lying in relation to pancreas head? 2nd part of duodenum,bile duct
6. While operating for hyper PTH, surgeon finds 2 supr and 1 infr lobe, to locate the ectopic lobe she has to trace which artery? maybe inferior parathyroid artery

7. Histamine is predominantly released from ?? mast cells

8. Pt has loss of afferent limb of papillary reflex, asked to identify the structure in a cut section aferent limb of pupillary reflex is via optic nerve...so that must be the structure asked to identify

9. A neonate is for corrective cardiac surgery, to reach PDA catheter passed from femoral vein must pass throu--?Lpulm art, LA,RV,RA
femoral vein-rt atria-pulm trunk-left pulm artery-PDA

10. Pt has spastic paralysis, asked to locate the area responsible on a gross pic of brain
spastic paralysis is due to UMN lesion..so maybe the pyramidal tract...or higher areas in motor or premotor cortex
11. If sup mesenteric art is occluded at its origin and no sig collaterals, then which organ is most affected ?duo, ileum, caecum, colon

12. Pt with seizures and anosmia has which cerebral lobe affected anosmia indicates involvement of pyriform cortex( responsible for smell)....this is located in cerebellum...but quesasks which cerebral lobe involved

13. Hypertensive pt with head injury in lucid, later deteriorates brain CT given, asked if it is epidural, intra cereb, .. lucid interval indicates that its epidural

14. Asked for nerve supply to ant ½ of ext ear canal
ant 1/2 of ext ear canal-supplied by auriculotemporal n
post 1/2 by auricular br of vagus
facial surface of upper part of auricle-auriculotemporal
cranial surface of upper part of auricle-lesser occipitla n
both surfaces of lower part of auricle-great auricular nerve

15. A 21yr male with acute LLQ abd pain & fever with vomiting, what other signs will be present-tenderness at McBuneys, / pain on passive flexion of hip?

16. Pt with stone in parotid duct, asked for thro which muscle does duct pass to open in oral cavity- zygomaticus major, temporalis, masseter, buccinator, orb oris?

17. A druken pt sleeps on arm chair develops wrist drop- nerve ? radial nerve injury

18. 10 yr child with elbow dislocation would also have damage to-? Deep brachial art, median N, interosseous memb, brachial v median nerve injury

19. Microscopic pic of FT with ovum in the process of fertilization, plenty of sperms also seen around, asked to identify the struc derived from glycoproteins??

20. A girl with well dev breasts, pubic hair, has normal female ext with testes, neither male nor female int organs, what is the cause androgen insensitvity
Adie's Pupil Dilated pupil which may react better to near than to light. It is probably due to disease affecting the ciliary ganglion. Is hyper-sensitive to any weak Pilocarpine (eg. 0.1%) with constriction of the pupil, in contrast to the pharmacologically dilated pupil (eg. by atropine) which will not constrict. There may be an associated loss of tendon reflexes, particularly the ankle jerks, but there is almost never any associated systemic disease. Over a period of years, the condition is likely to become bilateral and the initially dilated pupil will gradually reduce in size. However, its poor reaction to light will continue.
2.- Marcus Gunn Pupil-paradoxical dilatation of pupils in swinging flash-light test,seen in retinal detachment,optic neuritis etc.
3.Pupils in pontine lesions-; pontine lesions cause miosis but normal light response. pin-point pupils following pontine haemorrhage;
4.central diencephalic herniation causes fixed dilated pupils:
5.Argyll Robertson Pupil- Accomodation Reflex Present(ARP-mnemonic).
6.Pupil in Uncal herniation- findings include ipsilateral pupillary dilation, loss of light reflex, and ptosis due to compression of cranial nerve III
Whch of d correct regarding
Wernikes Encephalopathy :

-Often develops into Korsakof's even when treated

-It has a mortality rate of 70 % to 80 % if untreated

-The treatment is thiamine 100 mg. PO daily for 5 days


-The symptoms include Nistagmus ataxia and
opisthotonos

-None of the above *

2. Which is the most serious complication of the supra
condilar fracture of the Humerus ?

-A compartment syndrome of forearm

-Failure to heal

-Healing in a non anatomic position

-Injury into the median nerve.

-Permanent restriction of the Elbow motion.

3. Which one of the following descriptors of a
diagnostic test is Influenced by the prevalence of the
disease being tested for :

-Specificity

-Sensitivity

-Accuracy

-Positive predictive value

-Reliability

4. A 43 years M admitted for Emergency Gastrectomey
present confusion on the 3rd post-Op day. complains of
lack of sleep due to cockroaches on the ceiling he is
noted to be flashed & tremelus by the nurses during
the day .. the most likely problem is :

-Post-Op Electrolites Inbalance

-Paranoid Schizophrenia

-Depression Psychosis

-Delerium Tremons

-Anoxic brain system

5. Pt. with known type II DM. treated with
Clorpropamide present to ED in comma .. blood sugar =
1 mmol / L. .. you give the Pt 1 ampule of D50W & the
Pt wakes up promptley . what is the next appropriate
management :

-Give another ampule then discharge.

-Give 2 ampules then discharge.

-Give another ampule then observe the pt for 6 h. in
the ED.

-Give one more ampule , admit the Pt. & start IV.
Glucose Infusion with frequent ongoing glucose
measurement

6. Pregnant 36 weeks with vaginal watery discharge ,
management :

-do sterile speculum exam

7. Barbiturate withdrawal = convulsion

8. Retrovarted uterus complain = asymptomatic

9. G5 P4 post-delivery, postpartum hemorrhage, most
likely cause is :

-Uterine atony

10. Anal skin tag in child associated with ,

-chronic Anal Fissure

11. Child with abdominal pain attacks , drowning his
legs up , mucouid bloody stool. Diagnosis ?

-Intussusception

12. 1st sign of foot gangrene is = rest pain

13. Pt. M pain in both lower limbs with week popletial
artery pulsation , management ?

transvertebral angiogram

14. basket ball player averted his ankle joint during
jumping at match , on Examination Pain with Increased
Various range. Management ?

-Repair ligament surgery

-Below knee cast

15. clean wound cut with laceration & incomplete
section of nerve , management ?

-Suture of wound Immediately

-Leave the wound open

16. pain on walking increase in 2nd & 3rd metatarsal
bone of forefoot. Diagnosis ?

-Stress fracture

-Sesamoid bone

17. Pt. 68 years going to elective surgery , he has 4
mo. history of chest pain & got same attack at night
before the day of surgery , what do u do ?

-Lignocaine before surgery

-Trinatreate during surgery

-Send him for ICU

-Postpone the operation & check function of the
coronaries

18. collage student F 20 years presented with lower
abdominal pain , PE bilateral lower abdominal
tenderness , febrile Vaginal exam : tender with
cervical mobilization , pregnant test (-). Diagnosis ?


-Acute salpengitis

-Ectopic pregnancy

-UTI

19. vaginal bleeding , stooped at the day of admission
, pregnancy test is (+) US shows empty uterus & left
adnexal mass. Diagnosis ?

-Ectopic pregnancy

20. young Pt with history of non productive cough but
clinically well .. C-X ray shows bilateral basal
Infiltration .. Diagnosis. ?

-Mycoplasma Pneumonia

21. Pt with Ulcer in sole of the foot , at 2nd & 3rd
metatarsal joint X Ray necrotic & destroyed part of
2nd & 3rd metatarsal bone .. what to do ?

-Radical debridment

-Above knee amputation

-Daily sterile dressing

-Oral antibiotic

22. Problem which cause most work days = back pain

23. Nicotinic skin patchy is contraindicated in:

-Pregnancy

-CVA

-Ischemic Heart disease

-Alcoholic

NB. (not sure)

24. Psychotherapy is superior to medication in:

-Schizophrenia

-Bipolar disorder

-Alcoholic withdrawal

-Dysthimia

25. mother worried about her child because of history
of myopathy in family .. what Investigation to be done
1st ?

-CPK

-Muscle biopsy

-Nerve biopsy

-EMG

26. Pt on Lithium therapy became weak, lethargic,
Intolerant to heat. what Investigation must be done ?

-Lithium level

-BP

-Thyroid Function test

27. surgical nurse 25 years old is concerned that she
is loosing her mind, for the past 6 months she is been
preoccupied with contamination, on his ward she has
been not touching any patient ,door knobs ...etc.

She was also washing her hand excessively, which of
the following treatments will help in reducing her
preoccupation and hand washing ?

-Fluxetine

-Lorazepam

-Perphenazine

-Insight-oriented psychotherapy

-Nifedipine

28. M 50 years with Prostatic Cancer with bone
metastasis. Treatment ?

radical prostatectomy

-Radiotherapy

-Hormonal therapy in the form of total androgenic
blockage

-IV chemotherapy

29. Pt with family history of urticaria, presented
with urticaria ... investigation revealed C1 esterase
inhibitor deficiency ... Diagnosis. ?

-Hereditary Angioedema

30. thickened upper Lt. lip with mild vesicles ...
Diagnosis ?

-Urticaria

-Angioedema

-HSV infection

31. Erithematous Vulva with whitish lesion of Candida
of Groin & Satellite lesions .. what is the
predisposing disease ?

-DM

-CA. vulva

-Lichen sclerosis

32. Pt with sudden cough & pain in the Rt. side of the
chest .. you suspect aspiration of foreign body ..
what finding of the C-X ray ?

-Hyperlucent Rt. side

33. Defrentiaion between Maxilary & Zigomatic
fractures ???????

34. calculation of the effectiveness of a vaccine

Incidence Non Vaccinate - Incidence Vaccinated

---------------------------------------------------------------
x 100

Incidence Non Vaccinate

35. child chalking during eating .. he became
Cyanotic, agitated, with forceful breathing .. 1st
action to do is :

-Hit him from his back while head downward

-Introduce your finger in his mouth

36. 17 years F. at stage 3 Tanner still not menses ..
management ?

-Examination of the pelvis

-Chromosome analysis

-Estrogen Progesterone level

-Reassurance

37. asthmatic Pt. presented in the ER with Dyspnea ,
used to inhaled salbutamol with no Improvement.. PE
shows dyspnea slit rhonchi at the upper chest & neck
... management. ?

-IV. fluid + Aminophine

-Coricosteroids Inhaled

-Inhaled Salbutamol + IV hydrocortisone + IV Fluid

38. Alfa fetoprotein Increased in

-Menengomyelocele

-Renal Agenesis

-Down Syndrome

39. pt with abnormal pap smear .. what is the next
step ?

-Colposcopy

40. 60 years F. presented with 5 x 5 cm. adnexal mass
.. management. ?

-Surgery

-Chemotherapy

-Radiotherapy

41. Pt. found unconscious on the floor behind the bar,
alcoholic, ER agitated semiconscious ... PE shows
laceration on the head, all the limbs can be moved,
all the others are normal?what is your action?

-CT of the head

-IV. fluid + Observation

-Discharge

42. Pt. fail down from the 2nd floor on his back ..
the following will be part of the Initial management
EXCEPT :

-Cervical collar

-IV. fluid

-Spine X Ray

-Urinary c


0 - ArchivalUser - 09-22-2008

one of u guys send me this

excellent site for different heart sound and ecg

check it out


http://www.blaufuss.org/




thanks to maxima, he send me this

1)http://library.med.utah.edu/WebPath/HISTHTML/NEURANAT/CNS014A.html
2) Asked a simple nerve lesion...and they explained the symptoms...some make sure you know this picture inside out.
http://library.med.utah.edu/WebPath/HISTHTML/NEURANAT/CNS363A.html
3) Must know slide
http://library.med.utah.edu/WebPath/HISTHTML/NEURANAT/CNS016A.html
4) Any slides like these..are a must know...they give it like anything. Click on the gross picture for the same one too.
http://library.med.utah.edu/WebPath/HISTHTML/NEURANAT/MRIFS02.html
5) another one
http://library.med.utah.edu/WebPath/HISTHTML/NEURANAT/MRIFS06.html
6) Another must know slide...you'll atleast get 2-3 questions on these, the MRI kinda sucks for this one..but check it out, see if you make out anything
http://library.med.utah.edu/WebPath/HISTHTML/NEURANAT/MRIFS06.html
http://library.med.utah.edu/WebPath/HISTHTML/NEURANAT/MRIT1C21.html
actually the MRIs for this is very crappy!
7) same sorta slides
http://library.med.utah.edu/WebPath/HISTHTML/NEURANAT/CNS274A.html
http://library.med.utah.edu/WebPath/HISTHTML/NEURANAT/CNS263A.html
8) KNow where the Amygdala is
http://library.med.utah.edu/WebPath/HISTHTML/NEURANAT/CNS271A.html
9) They've given a similar slide to this and tested it.
http://library.med.utah.edu/WebPath/HISTHTML/NEURANAT/CNS229A.html
10) Vertebral artery-Basilar artery...about ya know, the sydromes
http://library.med.utah.edu/WebPath/HISTHTML/NEURANAT/CNS316A.html
http://library.med.utah.edu/WebPath/HISTHTML/NEURANAT/CNS375A.html





0 - ArchivalUser - 09-22-2008

#330108
irfanmir - 08/21/08 12:05

http://www.heartandmetabolism.org/images/HM25/hm25crfig1.gif
http://www.anaesthetist.com/icu/organs/heart/ecg/images/e_vflut.jpg
http://kcsun3.tripod.com/6b40c830.gif
http://www.ambulancetechnicianstudy.co.uk/images/VF.gif
http://myweb.lsbu.ac.uk/dirt/museum/margaret/731-1435-2100360.jpg
http://www.hawaii.edu/medicine/pediatrics/pemxray/v2c02e.jpg + http://www.hawaii.edu/medicine/pediatrics/pemxray/v2c02d.jpg (normal)
http://www.vetmed.wsu.edu/MedSci520/images/neoplasia/n3.jpg + http://farm1.static.flickr.com/48/119271511_33f2336e56.jpg
http://www.breastpath.com/photos/FCC1.jpg + http://www.breastpath.com/photos/fibrocystic-1-500x.jpg
http://depts.washington.edu/physdx/images/club2.jpg
http://www.nlm.nih.gov/medlineplus/ency/images/ency/fullsize/2008.jpg + http://farm4.static.flickr.com/3213/2522279311_274500d914.jpg?v=0


what is the DX and association ?

irfanmir - 08/21/08 21:18

right answers are

1. MI -- ST segment elevation in lead II, III, AVF and V2 ST segment depression. (this perticular pt ihas Diabetes.
2.Ventricular flutter
3.
4. Ventricular fibrillation
5. Doudenal atresia -- dialated stomach and part of doudenum before obstruction show massive gas. absence of gas after atresia --- present with bilious vomiting.
6. Thymus hypoplasia -- Digeorge syndrome (3rd & 4th brachial arch defect ) . T cell def and tetany. Abnormality of ear, mandible and aortic arch. + Right CXR is normal, arrow show space filled with media stinal tissue: In adult it disappear with lung tissue and media stinum move upward. In adult its oblitration indcates R ventricular enlarge ment or media stinal mass.
7. Leiomyoma -- most common benign tumor of uterus.
8. Fibrocystic disease of breast -- most common disorder of breast, fibrosis, blue cyst & apocrine metaplasia. Risk of cancer with epithelial atypia. (B/w age 25 to 50)
9. Clubbing of nails -- occur due to chronic low blood oxygen level (associated with cystic fibrosis, cardio vascular and pulmonary conditions)
10. Koilonychia (spooning of nails) -- associated with Iron def, psoriasis, infection under the nail and chemo therapy.



sweetiful - 08/24/08 12:10

please can anybody clarify how to differentiate betweem venticular fibbrillation and flutter......look at images 2 and 4
Report Abuse



* Re:Conceptual Q ------------------------------ PIC
#1431165
klm25 - 08/24/08 13:40

in flutter u see the same pattern where as in fib the pattern chnges as u see in slide thats how i think it is.

Report Abuse



* Re:Conceptual Q ------------------------------ PIC
#1431167
klm25 - 08/24/08 13:41

i mean u dont see ups and downs in flutter


killmle - 08/21/08 22:27

Hi, thanks for posting all these great slides.

I have doubt regarding slide 6......if its thymic hypoplasia than it should not show obliteration of antr mediastinum ( as the thymus is almost absent or absent) as you explained in your explaination.
If its thymic related than in adults it should be -thymoma.

Can you pls clarify. Thanks


irfanmir - 08/21/08 22:32

i think u missunderstood what i said.

left or 1st slide is absent thymus. right or 2nd slide is normal kid as it shows shade of normal tissue. put sides side by side and compare.

about adults if it oblitatrate it could be mass as u said thymoma , lung tumor, right ventricular enlargement.

i hope this clarify things.



0 - ArchivalUser - 09-22-2008

hi
thankyou very much for all the above stuff,we really need that.
can u plz send me nbme forms1-6 with answer key.i really need them.my email, adress is dr_asmaamer@yahoo.ca
thanks and have all the best in future



0 - ArchivalUser - 09-22-2008

drtilakpasala - 08/22/08 12:38

couldnt resist to post this....

http://www3.interscience.wiley.com:8100/legacy/college/boyer/0471661791/anima...ication.swf

'learnt today'

DNA gyrase (topoisomerase II) literally nicks the BOTH the DNA strands into 2 and relaxes the positive supercoiling.

In prokaryotes....inhibited by quinolones
In eukaryotes....inhibited by etoposide (used in testicular ca)


#329811
irfanmir - 08/20/08 12:25

http://library.med.utah.edu/WebPath/COW/COW099.jpg
http://www.vet.uga.edu/vpp/clerk/reynolds/Figure%201.jpg
http://botit.botany.wisc.edu/toms_fungi/images/hcap1.jpg
http://aapredbook.aappublications.org/week/009_12.jpg + http://www.mevis-research.de/~hhj/Lunge/ima/InfAspThA66.JPG
http://bjr.birjournals.org/content/vol78/issue931/images/large/BJR57...811-18.jpeg
http://radiology.rsnajnls.org/content/vol216/issue2/images/large/r00a...u18g2x.jpeg
http://radiology.rsnajnls.org/content/vol216/issue2/images/large/r00a...u18g1x.jpeg
http://64.202.120.86/upload/image/news/innate-immunity-to-cancer/nk-cell.jpg
http://www.uphs.upenn.edu/ortho/oj/2002/html/PICS/p71f3.jpg
http://www.wheelessonline.com/image2/i1/eog3.jpg
http://www.owlnet.rice.edu/~dhruv/bloodreport_files/image043.jpg
http://www.residentandstaff.com/content/RSP/2006/11/img/RSP200611f21.jpg


what is DX and association ?


irfanmir - 08/20/08 21:28

right answers are

1. Pneumocystis carinii (gomori methamine silver stain) -- pneumonia in AIDS pt.
2. Cryptococcus neoformans (arrow indicate budding. ability to form brown black pigment on medium) -- Cause Meningitis in AIDS pt.
3. Histoplasma Capsulatum (Ohio and missisippi) present in soil contaminated with bird and bat dropping. Cause Respiratory Infection
4. Aspergillus -- Fungal ball in lung in immunocompromised pt.
5. Shagreen patch (tuberous sclerosis)
6. steeple sign (inverted V looking or narrow Epiglotis) --- seen in Croups
7. Normal Epiglotis
8. Natural Killer Cell -- (identification marker CD 16, 56) beside indepandant killing of microbe also kill cancer cell. Also kill microbe by Type II ADCC.
9. Birbeck Granules - Tennis racket shaped structures found in Langerhan's cells in histiocytosis X (Langerhan's cell histiocytosis) which includes eosinophilic granuloma, Letterer-Siwe disease and Hand-Schuller-Christian disease
10. Eosinophilic granuloma - most common in children

irfanmir - 08/20/08 21:39

yeah slide 4 = 2 links one is aspergillus and the + nxt is fungal ball cause by aspergillus.

irfanmir - 08/21/08 00:17

sorry i didnt noticed that i answered only 10.
ok there u go rest of 11 12


11. Rouleaux formation -- staking of RBC occur when plasma protein conc is high becase of them ESR also increase.
12. Palpable purpura -- Henoch-Schönlein purpura (systemic vasculitis, IgA deposition in skin & kidney) mainly occur in children (Triad of prupura, artheritis, abdominal pain)






0 - ArchivalUser - 09-22-2008

irfanmir - 08/18/08 12:42

http://mediswww.meds.cwru.edu/ecsample/yeartwo/pulmonary/images/section2/...mcr067b.jpg ---- this section is from liver lung
http://library.med.utah.edu/WebPath/jpeg4/LIVER041.jpg
http://pathmicro.med.sc.edu/ghaffar/gvh-pic.jpg ---------------- pt who recieved bone marrow transplantation recently
http://www.slh.wisc.edu/cytogenetics/cases/gifs/com_karyotypes/CoMNov...97karyo.gif
http://cache.eb.com/eb/image?id=59346&rendTypeId=4
http://upload.wikimedia.org/wikipedia/commons/4/4c/Psammoma.jpg
http://www.learningradiology.com/caseofweek/caseoftheweekpix2007-1/co...w279arr.jpg
http://bonetumor.org/images/brodietibap.jpg
http://bonetumor.org/tumors/images/adax1.jpg
http://pathology.ouhsc.edu/deptlabs/Alzheimer/Images/p121ar.jpg


what is the dx and association ?


irfanmir - 08/18/08 20:44

right answers

1. Heart failure cells -- hemosidren laden macrophages
2. nutmag liver ----- chronic passive conjestion of liver
3. Graft vs host disease
4. XYY syndrome --- common karyotype among criminals.
5. Fragile X syndrome -- mild to modrate mental retardation in males. female are carrier
6. Psammoma Body -- seen in papillary carcinoma (thyroid gland), meningioma, Serous cystadenocarcinoma (ovary), mesothelioma.
7. Onion skin layer on bone -- Ewing sarcoma.
8. Brodies abscess ( gray triangular mark ) --- pyogenic osteomyelitis
9. Soap bubble appearance on bone -- Giant cell tumor ( usually occur on tibia)
10. Hirano bodies (IC aggregates of actin and associated protiens) -- appear in Alzheimers, Pick dis, CFJ diseae and other neudegenerative disease.


OK

the slide # 7 is Ewing sarcoma i confirmed it. ( can u plz post some onion skin appearnace )

but #9 u r right i posted wrong link. (that one is Adamantinoma)

Down is the right link for Giant cell tumor (lytic lesion)
http://www.pathology.med.ohio-state.edu/paxit/deptbase/Paxit/Images/10561/P...AXIT026.JPG